🧠

PRML 第4章 解答例

2022/06/09に公開
10

はじめに

PRML解答例まとめを参照

演習 4.1

データ\{ \mathbf{x}_n\}の集合が与えられ,凸包(convex hull)とは以下の式で与えられるすべての点の集合であると定義することができる.

\mathbf{x} = \sum_{n}\alpha_n \mathbf{x}_n \tag{4.156}

ここで\alpha_n \geq 0であり,\sum_n \alpha_n = 1である.第2のデータ\{ \mathbf{y}_n\}の集合とそれに対応する凸包を考える.もしすべてのデータ\mathbf{x}_nに対し\hat{\mathbf{w}}^{\mathrm{T}}\mathbf{x}_n + w_0 > 0を満たし,すべてのデータ\mathbf{y}_nに対し\hat{\mathbf{w}}^{\mathrm{T}}\mathbf{y}_n + w_0 < 0を満たすベクトル\hat{\mathbf{w}}とスカラーw_0が存在するなら,定義によりこれら2つのデータの集合は線形分離可能である.それらの凸包が重なる場合,2つのデータの集合は線形分離可能ではないことを示せ.また,逆に2つのデータの集合が線形分離可能な場合,それらの凸包が重ならないことを示せ.


凸包 (Wikipedia)とは与えられた集合を含む最小の凸集合である。例えばXがユークリッド平面内の有界な点集合のとき、その凸包は直観的にはXをゴム膜で包んだときにゴム膜が作る図形として視認することができる。つまり「与えられたデータを包み込む最小の集合」なので、凸包が2つのデータ集合間で重なっている場合には、線形分離不可能であるイメージが直観的に理解できる。矛盾を示す際には、両方にまたがる交差領域を使う。

それらの凸包が重なる場合,2つのデータの集合は線形分離可能ではないことを示せ」について

凸包が重なるというのは、2つのデータ集合\{\mathbf{x}_n\}\{\mathbf{y}_m\}が存在し、それぞれについての凸包のintersect(交差)領域が存在することを意味する。そしてそのようなintersect上の点\mathbf{z}

\mathbf{z}=\sum_{n} \alpha_{n} \mathbf{x}_{n}=\sum_{m} \beta_{m} \mathbf{y}_{m}

のように書くことができる。ここで\beta_mはすべてのmについて\beta_m \geq 0であり、\sum_m \beta_m = 1を満たす。

ここで、凸包が重なる場合であっても2つのデータ集合\{\mathbf{x}_n\}\{\mathbf{y}_m\}が線形分離可能であると仮定する。このとき、2つのintersect上に存在する点\mathbf{z}について

\begin{aligned} \hat{\mathbf{w}}^{\mathrm T}\mathbf{z} + w_0 &= \sum_n \alpha_n \hat{\mathbf{w}}^{\mathrm T}\mathbf{x}_n + w_0 \\ &= \sum_n \alpha_n \hat{\mathbf{w}}^{\mathrm T}\mathbf{x}_n + \underbrace{\left( \sum_n\alpha_n \right)}_{1}w_0 \\ &= \sum_n \alpha_n \left( \hat{\mathbf{w}}^{\mathrm T}\mathbf{x}_n + w_0 \right) > 0 \quad (\because \hat{\mathbf{w}}^{\mathrm T}\mathbf{x}_n + w_0 > 0) \end{aligned}

一方で

\begin{aligned} \hat{\mathbf{w}}^{\mathrm T}\mathbf{z} + w_0 &= \sum_m \beta_m \hat{\mathbf{w}}^{\mathrm T}\mathbf{y}_m + w_0 \\ &= \sum_m \beta_m \hat{\mathbf{w}}^{\mathrm T}\mathbf{y}_m + \underbrace{\left( \sum_m \beta_m \right)}_{1}w_0 \\ &= \sum_m \beta_m \left( \hat{\mathbf{w}}^{\mathrm T}\mathbf{y}_m + w_0 \right) < 0 \quad (\because \hat{\mathbf{w}}^{\mathrm T}\mathbf{y}_m + w_0 < 0) \end{aligned}

となる。これは矛盾するので仮定は誤りであり、凸包が重なる場合には2つのデータ集合\{\mathbf{x}_n\}\{\mathbf{y}_m\}は線形分離可能ではないことが示された。また、この対偶を取ることで「2つのデータの集合が線形分離可能な場合,それらの凸包が重ならない」ことも自動的に示される。

演習 4.2

二乗和誤差関数

E_{D}(\widetilde{\mathbf{W}})=\frac{1}{2} \operatorname{Tr}\left\{(\widetilde{\mathbf{X}} \widetilde{\mathbf{W}}-\mathbf{T})^{\mathrm{T}}(\widetilde{\mathbf{X}} \widetilde{\mathbf{W}}-\mathbf{T})\right\} \tag{4.15}

の最小化を考える.学習データ集合におけるすべての目的変数ベクトルが線形制約

\mathbf{a}^{\mathrm{T}} \mathbf{t}_{n}+b=0 \tag{4.157}

を満たすと仮定する.ここで,\mathbf{t}_n^{\mathrm{T}}(4.15)における行列\mathbf{T}n番目の行に相当する.この制約の結果として,最小二乗解

\mathbf{y}(\mathbf{x})=\widetilde{\mathbf{W}}^{\mathrm{T}} \widetilde{\mathbf{x}}=\mathbf{T}^{\mathrm{T}}\left(\tilde{\mathbf{X}}^{\dagger}\right)^{\mathrm{T}}\widetilde{\mathbf{x}},\quad \text{where}\quad \tilde{\mathbf{X}}^{\dagger} = \left(\tilde{\mathbf{X}}^{\mathrm{T}} \tilde{\mathbf{X}}\right)^{-1} \tilde{\mathbf{X}}^{\mathrm{T}} \tag{4.17}

によって与えられるモデルの予測\mathbf{y}(\mathbf{x})の要素もまたこの制約を満たす,つまり,以下の式を満たすことを示せ.

\mathbf{a}^{\mathrm{T}} \mathbf{y}(\mathbf{x})+b=0 \tag{4.158}

上記を示すため,パラメータw_0がバイアスとしての役割を持つように,基底関数が\phi_0(\mathbf{x})=1であると仮定する.


まずそれぞれの行列の形を確認する。

\mathbf{\widetilde{X}}=\begin{pmatrix} 1 & \mathbf{x}_1^{\mathrm{T}} \\ 1 & \mathbf{x}_2^{\mathrm{T}} \\ \vdots & \vdots \\ 1 & \mathbf{x}_n^{\mathrm{T}} \\ \end{pmatrix} = \begin{pmatrix}\mathbf{1} & \mathbf{X}\end{pmatrix},\quad \mathbf{\widetilde{W}}=\begin{pmatrix} w_{10} & w_{20} & \cdots & w_{K0} \\ \mathbf{w}_1 & \mathbf{w}_2 & \cdots & \mathbf{w}_K \\ \end{pmatrix}= \begin{pmatrix}\mathbf{w}_0^{\mathrm{T}} \\ \mathbf{W}\end{pmatrix}

\mathbf{\widetilde{X}}N\times (D+1)行列, \mathbf{\widetilde{W}}(D+1)\times K行列である。\mathbf{w}_0はバイアスパラメータのベクトルである。また\mathbf{1}N次元の全要素が1の列ベクトルを表しており、ダミー入力x_0 = 1を明示的に表示するために入れている。

\mathbf{T}N\times K行列で

\mathbf{T}=\begin{pmatrix} \mathbf{t}_1^{\mathrm{T}} \\ \mathbf{t}_2^{\mathrm{T}} \\ \vdots \\ \mathbf{t}_n^{\mathrm{T}} \end{pmatrix}

と書ける。

これらからバイアスパラメータとダミー入力を分離して(4.15)の誤差二乗和関数E_D(\widetilde{\mathbf{W}})を表すと

\begin{aligned} E_{D}(\widetilde{\mathbf{W}})&=\frac{1}{2} \operatorname{Tr}\left\{(\widetilde{\mathbf{X}} \widetilde{\mathbf{W}}-\mathbf{T})^{\mathrm{T}}(\widetilde{\mathbf{X}} \widetilde{\mathbf{W}}-\mathbf{T})\right\} \\ &=\frac{1}{2} \operatorname{Tr}\left\{ (\mathbf{XW}+\mathbf{1}\mathbf{w}_0^{\mathrm{T}}-\mathbf{T})^{\mathrm{T}}(\mathbf{XW}+\mathbf{1}\mathbf{w}_0^{\mathrm{T}}-\mathbf{T}) \right\} \end{aligned}

となる。

E_D(\widetilde{\mathbf{W}})の最小化のためにまず\mathbf{w}_0について微分すると

\begin{aligned} \frac{\partial E_D}{\partial \mathbf{w}_0} &= \frac{1}{2}\cdot 2\left( \mathbf{XW}+\mathbf{1}\mathbf{w}_0^{\mathrm{T}}-\mathbf{T} \right)^{\mathrm{T}}\mathbf{1} \\ &=\left( \mathbf{XW} - \mathbf{T} \right)^{\mathrm{T}}\mathbf{1} + \mathbf{w}_0 \mathbf{1}^{\mathrm{T}}\mathbf{1} \\ &=\left( \mathbf{XW} - \mathbf{T} \right)^{\mathrm{T}}\mathbf{1} + N\mathbf{w}_0 \end{aligned}

これが\mathbf{0}となれば良いので

\begin{aligned} \mathbf{w}_0 &= \frac{1}{N}\left( \mathbf{T} - \mathbf{XW} \right)^{\mathrm{T}}\mathbf{1} \\ &= \frac{1}{N}\mathbf{T}^{\mathrm{T}}\mathbf{1} - \frac{1}{N}\mathbf{W}^{\mathrm{T}}\mathbf{X}^{\mathrm{T}}\mathbf{1} \\ &:= \overline{\mathbf{t}}-\mathbf{W}^{\mathrm{T}}\overline{\mathbf{x}} \end{aligned}

ここで、以降の略記のために\displaystyle \overline{\mathbf{t}} = \frac{1}{N}\mathbf{T}^{\mathrm{T}}\mathbf{1},\quad \overline{\mathbf{x}} = \frac{1}{N}\mathbf{X}^{\mathrm{T}}\mathbf{1}とした。

E_D(\widetilde{\mathbf{W}})\mathbf{w}_0をこれで書き換えると

\begin{aligned} E_{D}(\mathbf{W}) &= \frac{1}{2} \operatorname{Tr}\left\{(\mathbf{X} \mathbf{W}+\overline{\mathbf{T}}-\overline{\mathbf{X}} \mathbf{W}-\mathbf{T})^{\mathrm{T}}(\mathbf{X} \mathbf{W}+\overline{\mathbf{T}}-\overline{\mathbf{X}} \mathbf{W}-\mathbf{T})\right\} \\ &= \frac{1}{2} \operatorname{Tr}\left\{\left((\mathbf{X} - \overline{\mathbf{X}})\mathbf{W} - (\mathbf{T} - \overline{\mathbf{T}})\right)^{\mathrm{T}}\left((\mathbf{X} - \overline{\mathbf{X}})\mathbf{W} - (\mathbf{T} - \overline{\mathbf{T}})\right)\right\} \end{aligned}

ここで略記のために\displaystyle \overline{\mathbf{T}}=\mathbf{1} \overline{\mathbf{t}}^{\mathrm{T}}, \quad \overline{\mathbf{X}}=\mathbf{1} \overline{\mathbf{x}}^{\mathrm{T}}とした。

これについても\mathbf{W}について微分して\mathbf{0}をとると

\frac{\partial E_{D}}{\partial \mathbf{W}} = \frac{1}{2}\cdot 2 (\mathbf{X} - \overline{\mathbf{X}})^{\mathrm{T}}\left((\mathbf{X} - \overline{\mathbf{X}})\mathbf{W} - (\mathbf{T} - \overline{\mathbf{T}})\right) = \mathbf{0}

\widehat{\mathbf{X}}=\mathbf{X}-\overline{\mathbf{X}}, \widehat{\mathbf{T}}=\mathbf{T}-\overline{\mathbf{T}}と書き直すと

\mathbf{W}=\left(\widehat{\mathbf{X}}^{\mathrm{T}} \widehat{\mathbf{X}}\right)^{-1} \widehat{\mathbf{X}}^{\mathrm{T}} \widehat{\mathbf{T}}=\widehat{\mathbf{X}}^{\dagger} \widehat{\mathbf{T}}

となる。

以上から、新しい入力\mathbf{x}^{*}が得られたときの予測値\mathbf{y}(\mathbf{x}^{*})

\begin{aligned} \mathbf{y}\left(\mathbf{x}^{*}\right) &= \widetilde{\mathbf{W}}^{\mathrm{T}} \widetilde{\mathbf{x}^{*}}\\ &= \begin{pmatrix}\mathbf{w}_0 & \mathbf{W}^{\mathrm{T}}\end{pmatrix} \begin{pmatrix}1 \\ \mathbf{x}^{*}\end{pmatrix} \\ &=\mathbf{W}^{\mathrm{T}} \mathbf{x}^{\star}+\mathbf{w}_{0} \\ &=\mathbf{W}^{\mathrm{T}} \mathbf{x}^{\star}+\overline{\mathbf{t}}-\mathbf{W}^{\mathrm{T}} \overline{\mathbf{x}} \\ &=\overline{\mathbf{t}}+\widehat{\mathbf{T}}^{\mathrm{T}}\left(\widehat{\mathbf{X}}^{\dagger}\right)^{\mathrm{T}}\left(\mathbf{x}^{\star}-\overline{\mathbf{x}}\right) \end{aligned}

となる。

一方、(4.157)式の線形制約条件は\overline{\mathbf{t}}について適用すると、以下のように書き直せる

\begin{aligned} \mathbf{a}^{\mathrm{T}}\overline{\mathbf{t}} + b &= \frac{1}{N}(\mathbf{a}^{\mathrm{T}}\mathbf{T}^{\mathrm{T}}\mathbf{1} + Nb) \\ &= \frac{1}{N}\sum_{n=1}^N(\mathbf{a}^{\mathrm{T}}\mathbf{t}_n + b) \\ &= 0 \end{aligned}

すなわち\mathbf{a}^{\mathrm{T}}\overline{\mathbf{t}} = -bが得られる。また、\mathbf{a}^{\mathrm{T}}\mathbf{t}_n + b = 0\mathbf{T}^{\mathrm{T}}について適用すると

\mathbf{a}^{\mathrm{T}}\mathbf{T}^{\mathrm{T}} = \begin{pmatrix} -b & -b & \cdots & -b \end{pmatrix} = -b\mathbf{1}^{\mathrm{T}}

となる。

以上を用いて、(4.157)式の線形制約が成立しているとき、モデルの予測\mathbf{y}(\mathbf{x}^{*})について(4.158)の左辺を計算してみると

\begin{aligned} \mathbf{a}^{\mathrm{T}} \mathbf{y}\left(\mathbf{x}^{\star}\right) + b &=\mathbf{a}^{\mathrm{T}} \overline{\mathbf{t}}+\mathbf{a}^{\mathrm{T}} \widehat{\mathbf{T}}^{\mathrm{T}}\left(\hat{\mathbf{X}}^{\dagger}\right)^{\mathrm{T}}\left(\mathbf{x}^{\star}-\overline{\mathbf{x}}\right) + b \\ &= \mathbf{a}^{\mathrm{T}} \widehat{\mathbf{T}}^{\mathrm{T}}\left(\hat{\mathbf{X}}^{\dagger}\right)^{\mathrm{T}}\left(\mathbf{x}^{\star}-\overline{\mathbf{x}}\right) \\ &= \mathbf{a}^{\mathrm{T}} (\mathbf{T} - \overline{\mathbf{T}})^{\mathrm{T}}\left(\hat{\mathbf{X}}^{\dagger}\right)^{\mathrm{T}}\left(\mathbf{x}^{\star}-\overline{\mathbf{x}}\right) \\ &= (\mathbf{a}^{\mathrm{T}}\mathbf{T}^{\mathrm{T}} - \mathbf{a}^{\mathrm{T}}\overline{\mathbf{T}}^{\mathrm{T}})\left(\hat{\mathbf{X}}^{\dagger}\right)^{\mathrm{T}}\left(\mathbf{x}^{\star}-\overline{\mathbf{x}}\right) \\ &= (\mathbf{a}^{\mathrm{T}}\mathbf{T}^{\mathrm{T}} - \mathbf{a}^{\mathrm{T}}\overline{\mathbf{t}}\mathbf{1}^{\mathrm{T}})\left(\hat{\mathbf{X}}^{\dagger}\right)^{\mathrm{T}}\left(\mathbf{x}^{\star}-\overline{\mathbf{x}}\right) \\ &= (-b\mathbf{1}^{\mathrm{T}} + b\mathbf{1}^{\mathrm{T}})\left(\hat{\mathbf{X}}^{\dagger}\right)^{\mathrm{T}}\left(\mathbf{x}^{\star}-\overline{\mathbf{x}}\right) \\ &= \mathbf{0}^{\mathrm{T}} \left(\hat{\mathbf{X}}^{\dagger}\right)^{\mathrm{T}}\left(\mathbf{x}^{\star}-\overline{\mathbf{x}}\right) \\ &= 0 \end{aligned}

よって、(4.158)式の\mathbf{a}^{\mathrm{T}} \mathbf{y}\left(\mathbf{x}^{\star}\right) + b = 0が示された。

目標値の分布が超平面に載るという拘束条件下で誤差関数を最小二乗和にしてクラス分類すると、その予測値の分布も超平面上に載るということである。

演習 4.3

演習問題4.2の結果を拡張し,多次元線形制約が目的変数ベクトルによって同時に満たされる場合,同じ制約が線形モデルの最小二乗予測によっても満たされることを示せ.


演習問題4.2の仮定を拡張する。
「多次元線形制約が目的変数ベクトルによって同時に満たされる場合」というのは、(4.157)式の線形制約が

\mathbf{A}\mathbf{t}_n+\mathbf{b} = \mathbf{0}

となることを表す。ここで、\mathbf{A}は行列で\mathbf{b}はベクトルであり、\mathbf{A}\mathbf{b}の各行が1つの線形制約を表すようなものである。つまりlを多次元線形制約の次元数として

\mathbf{A} = \begin{pmatrix} \mathbf{a}_1^{\mathrm T} \\ \mathbf{a}_2^{\mathrm T} \\ \vdots \\ \mathbf{a}_l^{\mathrm T} \end{pmatrix}, \quad \mathbf{b} = \begin{pmatrix} b_1 \\ b_2 \\ \vdots \\ b_l \end{pmatrix}

となる。

題意「同じ制約が線形モデルの最小二乗予測によっても満たされることを示せ」は、\mathbf{A}\mathbf{t}_n+\mathbf{b} = \mathbf{0}が成り立つ中で\mathbf{A}\mathbf{y}(\mathbf{x}^{\star})+\mathbf{b}\mathbf{0}となることを示すことなので、これを演習問題4.2の最後の式に当てはめると

\begin{aligned} \mathbf{A}^{\mathrm{T}} \mathbf{y}\left(\mathbf{x}^{\star}\right) + \mathbf{b} &=\mathbf{A}^{\mathrm{T}} \overline{\mathbf{t}}+\mathbf{A}^{\mathrm{T}} \widehat{\mathbf{T}}^{\mathrm{T}}\left(\hat{\mathbf{X}}^{\dagger}\right)^{\mathrm{T}}\left(\mathbf{x}^{\star}-\overline{\mathbf{x}}\right) + \mathbf{b} \\ &= \mathbf{A}^{\mathrm{T}} \widehat{\mathbf{T}}^{\mathrm{T}}\left(\hat{\mathbf{X}}^{\dagger}\right)^{\mathrm{T}}\left(\mathbf{x}^{\star}-\overline{\mathbf{x}}\right) \\ &= \mathbf{A}^{\mathrm{T}} (\mathbf{T} - \overline{\mathbf{T}})^{\mathrm{T}}\left(\hat{\mathbf{X}}^{\dagger}\right)^{\mathrm{T}}\left(\mathbf{x}^{\star}-\overline{\mathbf{x}}\right) \\ &= (\mathbf{A}^{\mathrm{T}}\mathbf{T}^{\mathrm{T}} - \mathbf{A}^{\mathrm{T}}\overline{\mathbf{T}}^{\mathrm{T}})\left(\hat{\mathbf{X}}^{\dagger}\right)^{\mathrm{T}}\left(\mathbf{x}^{\star}-\overline{\mathbf{x}}\right) \\ &= (\mathbf{A}^{\mathrm{T}}\mathbf{T}^{\mathrm{T}} - \mathbf{A}^{\mathrm{T}}\overline{\mathbf{t}}\mathbf{1}^{\mathrm{T}})\left(\hat{\mathbf{X}}^{\dagger}\right)^{\mathrm{T}}\left(\mathbf{x}^{\star}-\overline{\mathbf{x}}\right) \\ &= (-\mathbf{b}\mathbf{1}^{\mathrm{T}} + \mathbf{b}\mathbf{1}^{\mathrm{T}})\left(\hat{\mathbf{X}}^{\dagger}\right)^{\mathrm{T}}\left(\mathbf{x}^{\star}-\overline{\mathbf{x}}\right) \\ &= \mathbf{0}^{\mathrm{T}} \left(\hat{\mathbf{X}}^{\dagger}\right)^{\mathrm{T}}\left(\mathbf{x}^{\star}-\overline{\mathbf{x}}\right) \\ &= \mathbf{0} \end{aligned}

よって\mathbf{A}\mathbf{y}(\mathbf{x}^{\star})+\mathbf{b} = \mathbf{0}となるので、題意が示された。

演習 4.4

制約\mathbf{w}^{\mathrm{T}}\mathbf{w}=1を満たすようにラグランジュ乗数を利用し,

m_{2}-m_{1}=\mathbf{w}^{\mathrm{T}}\left(\mathbf{m}_{2}-\mathbf{m}_{1}\right) \tag{4.22}

によって与えられるクラス分離規準を\mathbf{w}に関して最大化すれば\mathbf{w} \propto (\mathbf{m}_2-\mathbf{m}_1)となることを示せ.


\mathbf{w}^\mathrm{T}\mathbf{w} =1の制約の下でラグランジュの未定乗数法を用いると

\begin{aligned} L &= \mathbf{w}^\mathrm{T}( \mathbf{m}_2-\mathbf{m}_1)+\lambda(\mathbf{w}^\mathrm{T}\mathbf{w}-1) \\ \frac{\partial L}{\partial \mathbf{w}}&= \mathbf{m}_2-\mathbf{m}_1+2\lambda \mathbf{w} \end{aligned}

これが\mathbf{0}になる点を考えると

\mathbf{w} = -\frac{1}{2\lambda}(\mathbf{m}_2-\mathbf{m}_1)

となり\mathbf{w}\propto(\mathbf{m}_2-\mathbf{m}_1)であることが示せた。

演習 4.5

y = \mathbf{w}^{\mathrm T}\mathbf{x} \tag{4.20}
m_k = \mathbf{w}^{\mathrm T}\mathbf{m}_k \tag{4.23}

s_{k}^{2}=\sum_{n \in \mathcal{C}_{k}}\left(y_{n}-m_{k}\right)^{2} \tag{4.24}

を使って,フィッシャーの判別規準

J(\mathbf{w})=\frac{\left(m_{2}-m_{1}\right)^{2}}{s_{1}^{2}+s_{2}^{2}} \tag{4.25}

J(\mathbf{w})=\frac{\mathbf{w}^{\mathrm{T}} \mathbf{S}_{\mathrm{B}} \mathbf{w}}{\mathbf{w}^{\mathrm{T}} \mathbf{S}_{\mathrm{W}} \mathbf{w}} \tag{4.26}

の形で書けることを示せ.


\mathbf{S}_{\mathrm{B}}\mathbf{S}_{\mathrm{W}}の定義はそれぞれ(4.27)(4.28)で与えられる。

(4.27)(4.28)より

\begin{aligned} \mathbf{S}_{\mathrm{B}}&=\left(\mathbf{m}_{2}-\mathbf{m}_{1}\right)\left(\mathbf{m}_{2}-\mathbf{m}_{1}\right)^{\mathrm{T}} \\ \mathbf{S}_{\mathrm{W}}&=\sum_{n \in \mathcal{C}_{1}}\left(\mathbf{x}_{n}-\mathbf{m}_{1}\right)\left(\mathbf{x}_{n}-\mathbf{m}_{1}\right)^{\mathrm{T}}+\sum_{n \in \mathcal{C}_{2}}\left(\mathbf{x}_{n}-\mathbf{m}_{2}\right)\left(\mathbf{x}_{n}-\mathbf{m}_{2}\right)^{\mathrm{T}} \end{aligned}

である。これを用いて(4.25)を変形していくと

\begin{aligned} J(\mathbf{w}) &=\frac{\left(m_{2}-m_{1}\right)^{2}}{s_{1}^{2}+s_{2}^{2}} \\ &=\frac{\left(\mathbf{w}^{\mathrm{T}}\left(\mathbf{m}_{2}-\mathbf{m}_{1}\right)\right)^{2}}{\sum_{n \in \mathcal{C}_{1}}\left(\mathbf{w}^{\mathrm{T}}\left(\mathbf{x}_{n}-\mathbf{m}_{1}\right)\right)^{2}+\sum_{n \in \mathcal{C}_{2}}\left(\mathbf{w}^{\mathrm{T}}\left(\mathbf{x}_{n}-\mathbf{m}_{2}\right)\right)^{2}} \\ &=\frac{\left(\mathbf{w}^{\mathrm{T}}\left(\mathbf{m}_{2}-\mathbf{m}_{1}\right)\right)\left(\left(\mathbf{m}_{2}-\mathbf{m}_{1}\right)^{\mathrm{T}} \mathbf{w}\right)}{\sum_{n \in \mathcal{C}_{1}}\left(\mathbf{w}^{\mathrm{T}}\left(\mathbf{x}_{n}-\mathbf{m}_{1}\right)\left(\mathbf{x}_{n}-\mathbf{m}_{1}\right)^{\mathrm{T}} \mathbf{w}\right)+\sum_{n \in \mathcal{C}_{2}}\left(\mathbf{w}^{\mathrm{T}}\left(\mathbf{x}_{n}-\mathbf{m}_{2}\right)\left(\mathbf{x}_{n}-\mathbf{m}_{2}\right)^{\mathrm{T}} \mathbf{w}\right)} \\ &=\frac{\mathbf{w}^{\mathrm{T}} \mathbf{S}_{\mathrm B} \mathbf{w}}{\mathbf{w}^{\mathrm{T}}\left\{\sum_{n \in \mathcal{C}_{1}}\left(\mathbf{x}_{n}-\mathbf{m}_{1}\right)\left(\mathbf{x}_{n}-\mathbf{m}_{1}\right)^{\mathrm{T}}+\sum_{n \in \mathcal{C}_{2}}\left(\mathbf{x}_{n}-\mathbf{m}_{2}\right)\left(\mathbf{x}_{n}-\mathbf{m}_{2}\right)^{\mathrm{T}}\right\} \mathbf{w}} \\ &=\frac{\mathbf{w}^{\mathrm{T}} \mathbf{S}_{\mathrm B} \mathbf{w}}{\mathbf{w}^{\mathrm{T}} \mathbf{S}_{\mathrm W} \mathbf{w}} \end{aligned}

となり(4.26)式が得られる。


(4.29)の導出について

J(\mathbf{w})=\left(\mathbf{w}^{\mathrm{T}} \mathbf{S}_{\mathrm{B}} \mathbf{w}\right)\left(\mathbf{w}^{\mathrm{T}} \mathbf{S}_{\mathrm{W}} \mathbf{w}\right)^{-1} \tag{4.26}

より、ここで

\frac{\partial}{\partial \mathbf{w}}\left(\mathbf{w}^{\mathrm{T}} \mathbf{S}_{\mathrm{B}} \mathbf{w}\right)=2 \mathbf{S}_{\mathrm{B}} \mathbf{w}, \frac{\partial}{\partial \mathbf{w}}\left(\mathbf{w}^{\mathrm{T}} \mathbf{S}_{\mathrm{B}} \mathbf{w}\right)=2 \mathbf{S}_{\mathrm{W}} \mathbf{w}

\mathbf{w}^{\mathrm{T}} \mathbf{S}_{\mathrm{B}} \mathbf{w}=\mathbf{\Lambda}, \mathbf{w}^{\mathrm{T}} \mathbf{S}_{\mathrm{W}} \mathbf{w}=\mathbf{L}とおくと

\begin{aligned} \frac{\partial J(\mathbf{w})}{\partial \mathbf{w}}&=\frac{\partial}{\partial \mathbf{w}}\left(\mathbf{\Lambda} \mathbf{L}^{-1}\right)=\frac{\partial}{\partial \mathbf{w}} \mathbf{\Lambda} \mathbf{L}^{-1}+\mathbf{\Lambda} \frac{\partial}{\partial \mathbf{w}} \mathbf{L}^{-1} \quad (\because (\mathrm{C}.20)) \\ & =2 \mathbf{S}_{\mathrm{B}} \mathbf{w} \mathbf{L}^{-1}-\mathbf{\Lambda} \mathbf{L}^{-1} \frac{\partial \mathbf{L}}{\partial \mathbf{w}} \mathbf{L}^{-1} \quad (\because (\mathrm{C}.21)) \\ & =2 \mathbf{S}_{\mathrm{B}} \mathbf{w} \mathbf{L}^{-1}-2 \mathbf{\Lambda} \mathbf{L}^{-1} \mathbf{S}_{\mathrm{W}} \mathbf{w} \mathbf{L}^{-1} \end{aligned}

これを0とおくと, \mathbf{\Lambda}\mathbf{L}を左や右からかけたりして(※\mathbf{\Lambda}\mathbf{L}はいずれもスカラー値になるはずなので、辺の左端に移動させてもよい……はず)

\begin{aligned} \mathbf{S}_{\mathrm{B}} \mathbf{w} \mathbf{L}^{-1}&=\mathbf{\Lambda} \mathbf{L}^{-1} \mathbf{S}_{\mathrm{W}} \mathbf{w} \mathbf{L}^{-1} \\ \mathbf{S}_{\mathrm{B}} \mathbf{w} &= \mathbf{\Lambda}\mathbf{L}^{-1} \mathbf{S}_{\mathrm{W}} \mathbf{w} \\ \mathbf{\Lambda}^{-1} \mathbf{S}_{\mathrm{B}} \mathbf{w}&=\mathbf{L}^{-1} \mathbf{S}_{\mathrm{W}} \mathbf{w} \\ \mathbf{S}_{\mathrm{B}} \mathbf{w}&=\mathbf{L}^{-1}\mathbf{\Lambda} \mathbf{S}_{\mathrm{W}} \mathbf{w} \quad (\because \mathbf{L}^{-1}はスカラー)\\ \mathbf{L} \mathbf{S}_{\mathrm{B}} \mathbf{w}&=\mathbf{\Lambda} \mathbf{S}_{\mathrm{W}} \mathbf{w}\\ \left(\mathbf{w}^{\mathrm{T}} \mathbf{S}_{\mathrm{W}} \mathbf{w}\right) \mathbf{S}_{\mathrm{B}} \mathbf{w}&=\left(\mathbf{w}^{\mathrm{T}} \mathbf{S}_{\mathrm{B}} \mathbf{w}\right) \mathbf{S}_{\mathrm{W}} \mathbf{w} \quad \cdots (4.29) \end{aligned}

演習 4.6

\mathbf{S}_{\mathbf{B}}=\left(\mathbf{m}_{2}-\mathbf{m}_{1}\right)\left(\mathbf{m}_{2}-\mathbf{m}_{1}\right)^{\mathrm{T}} \tag{4.27}

で与えられるクラス間共分散行列と

\mathbf{S}_{\mathbf{W}}=\sum_{n \in \mathcal{C}_{1}}\left(\mathbf{x}_{n}-\mathbf{m}_{1}\right)\left(\mathbf{x}_{n}-\mathbf{m}_{1}\right)^{\mathrm{T}}+\sum_{n \in \mathcal{C}_{2}}\left(\mathbf{x}_{n}-\mathbf{m}_{2}\right)\left(\mathbf{x}_{n}-\mathbf{m}_{2}\right)^{\mathrm{T}} \tag{4.28}

で与えられるクラス内共分散行列のそれぞれの定義と,

w_0 = -\mathbf{w}^{\mathrm T}\mathbf{m} \tag{4.34}
\mathbf{m}=\frac{1}{N} \sum_{n=1}^{N} \mathbf{x}_{n}=\frac{1}{N}\left(N_{1} \mathbf{m}_{1}+N_{2} \mathbf{m}_{2}\right) \tag{4.36}

および4.1.5節で述べた目的値を使って,二乗和誤差関数を最小化する表現

\sum_{n=1}^{N}\left(\mathbf{w}^{\mathrm{T}} \mathbf{x}_{n}+w_{0}-t_{n}\right) \mathbf{x}_{n}=0 \tag{4.33}

\left(\mathbf{S}_{\mathbf{W}}+\frac{N_{1} N_{2}}{N} \mathbf{S}_{\mathbf{B}}\right) \mathbf{w}=N\left(\mathbf{m}_{1}-\mathbf{m}_{2}\right) \tag{4.37}

の形で書けることを示せ.


※ 簡単な代数演算をして……と書かれてあるが、計算量的には簡単じゃない。まず\displaystyle \sum_{n=1}^N t_n\mathbf{x}_n = N(\mathbf{m}_1-\mathbf{m}_2)となることを利用して、残りの部分で(\cdot)\mathbf{w}の式に書き直せるようにスカラー部分とベクトル部分を入れ替える技((\mathbf{a}^{\mathrm{T}}\mathbf{b})\mathbf{c} = \mathbf{c}\mathbf{b}^{\mathrm{T}}\mathbf{a})を使う。

まず(4.33)\displaystyle \sum_{n=1}^N t_n\mathbf{x}_nについて変形すると

\begin{aligned} \sum_{n=1}^N t_n\mathbf{x}_n &= \frac{N}{N_1}\sum_{n \in \mathcal{C}_1}\mathbf{x}_n - \frac{N}{N_2}\sum_{n \in \mathcal{C}_2}\mathbf{x}_n \\ &=\frac{N}{N_1}(N_1\mathbf{m}_1)-\frac{N}{N_2}(N_2\mathbf{m}_2) \\ &=N(\mathbf{m}_1 - \mathbf{m}_2) \end{aligned}

となる。これは(4.37)式の右辺になっているので、

\sum_{n=1}^{N}\left(\mathbf{w}^{\mathrm{T}} \mathbf{x}_{n}+w_{0}\right)\mathbf{x}_{n} = \left(\mathbf{S}_{\mathrm{W}}+\frac{N_{1} N_{2}}{N} \mathbf{S}_{\mathrm{B}}\right) \mathbf{w}

となることを示せれば良い。

\begin{aligned} \sum_{n=1}^{N}\left(\mathbf{w}^{\mathrm{T}} \mathbf{x}_{n}+w_{0}\right) \mathbf{x}_{n}&=\sum_{n=1}^{N}\left(\mathbf{w}^{\mathrm{T}} \mathbf{x}_{n}-\mathbf{w}^{\mathrm{T}} \mathbf{m}\right) \mathbf{x}_{n} \\ &=\sum_{n=1}^{N}\left\{\left(\mathbf{x}_{n} \mathbf{x}_{n}^{\mathrm{T}}-\mathbf{x}_{n} \mathbf{m}^{\mathrm{T}}\right) \mathbf{w}\right\} \\ &= \sum_{n \in \mathcal{C}_{1}}\left\{\left(\mathbf{x}_{n} \mathbf{x}_{n}^{\mathrm{T}}-\mathbf{x}_{n} \mathbf{m}^{\mathrm{T}}\right) \mathbf{w}\right\} + \sum_{m \in \mathcal{C}_{2}}\left\{\left(\mathbf{x}_{m} \mathbf{x}_{m}^{\mathrm{T}}-\mathbf{x}_{m} \mathbf{m}^{\mathrm{T}}\right) \mathbf{w}\right\} \\ &=\left(\sum_{n \in \mathcal{C}_{1}} \mathbf{x}_{n} \mathbf{x}_{n}^{\mathrm{T}}-N_{1} \mathbf{m}_{1} \mathbf{m}^{\mathrm{T}}\right) \mathbf{w} + \left(\sum_{m \in \mathcal{C}_{2}} \mathbf{x}_{m} \mathbf{x}_{m}^{\mathrm{T}}-N_{2} \mathbf{m}_{2} \mathbf{m}^{\mathrm{T}}\right) \mathbf{w} \\ &=\left(\sum_{n \in \mathcal{C}_{1}} \mathbf{x}_{n} \mathbf{x}_{n}^{\mathrm{T}}+\sum_{m \in \mathcal{C}_{2}} \mathbf{x}_{m} \mathbf{x}_{m}^{\mathrm{T}}-\left(N_{1} \mathbf{m}_{1}+N_{2} \mathbf{m}_{2}\right) \mathbf{m}^{\mathrm{T}}\right) \mathbf{w} \end{aligned}

となる。

ここで、(4.27)(4.28)の展開した形を用意しておく。(4.27)

\begin{aligned} \mathbf{S}_{\mathrm{B}} &= \left(\mathbf{m}_{2}-\mathbf{m}_{1}\right)\left(\mathbf{m}_{2}-\mathbf{m}_{1}\right)^{\mathrm{T}} \\ &=\mathbf{m}_2\mathbf{m}_2^{\mathrm{T}}-\mathbf{m}_2\mathbf{m}_1^{\mathrm{T}}-\mathbf{m}_1\mathbf{m}_2^{\mathrm{T}}+\mathbf{m}_1\mathbf{m}_1^{\mathrm{T}} \end{aligned}

となり、(4.28)

\begin{aligned} \mathbf{S}_{\mathrm W} = \sum_{k=1}^2 \sum_{i \in \mathcal{C}_{k}}\left(\mathbf{x}_{i}-\mathbf{m}_{k}\right)\left(\mathbf{x}_{i}-\mathbf{m}_{k}\right)^{\mathrm{T}} &= \sum_{k=1}^2 \sum_{i \in \mathcal{C}_{k}}\left(\mathbf{x}_{i} \mathbf{x}_{i}^{\mathrm{T}}-\mathbf{x}_{i} \mathbf{m}_{k}^{\mathrm{T}}-\mathbf{m}_{k} \mathbf{x}_{i}^{\mathrm{T}}+\mathbf{m}_{k} \mathbf{m}_{k}^{\mathrm{T}}\right) \\ &=\sum_{k=1}^2 \left( \sum_{i \in \mathcal{C}_{k}} \mathbf{x}_{i} \mathbf{x}_{i}^{\mathrm{T}}-N_{k} \mathbf{m}_{k} \mathbf{m}_{k}^{\mathrm{T}}\right) \\ &=\sum_{n \in \mathcal{C}_{1}} \mathbf{x}_{n} \mathbf{x}_{n}^{\mathrm{T}} + \sum_{m \in \mathcal{C}_{2}} \mathbf{x}_{m} \mathbf{x}_{m}^{\mathrm{T}} - N_1\mathbf{m}_{1} \mathbf{m}_{1}^{\mathrm{T}} - N_2\mathbf{m}_{2} \mathbf{m}_{2}^{\mathrm{T}} \end{aligned}

であるから、これらを利用して書き直すと

\begin{aligned} &\quad \left(\sum_{n \in \mathcal{C}_{1}} \mathbf{x}_{n} \mathbf{x}_{n}^{\mathrm{T}}+\sum_{m \in \mathcal{C}_{2}} \mathbf{x}_{m} \mathbf{x}_{m}^{\mathrm{T}}-\left(N_{1} \mathbf{m}_{1}+N_{2} \mathbf{m}_{2}\right) \mathbf{m}^{\mathrm{T}}\right) \mathbf{w} \\ &= \left(\mathbf{S}_{\mathbf{w}}+N_{1} \mathbf{m}_{1} \mathbf{m}_{1}^{\mathrm{T}}+N_{2} \mathbf{m}_{2} \mathbf{m}_{2}^{\mathrm{T}}-\left(N_{1} \mathbf{m}_{1}+N_{2} \mathbf{m}_{2}\right) \frac{1}{N}\left(N_{1} \mathbf{m}_{1}+N_{2} \mathbf{m}_{2}\right)^{\mathrm{T}}\right) \mathbf{w} \\ &= \left(\mathbf{S}_{\mathbf{w}}+\left(N_{1}-\frac{N_{1}^{2}}{N}\right) \mathbf{m}_{1} \mathbf{m}_{1}^{\mathrm{T}}-\frac{N_{1} N_{2}}{N}\left(\mathbf{m}_{1} \mathbf{m}_{2}^{\mathrm{T}}+\mathbf{m}_{2} \mathbf{m}_{1}^{\mathrm{T}}\right) +\left(N_{2}-\frac{N_{2}^{2}}{N}\right) \mathbf{m}_{2} \mathbf{m}_{2}^{\mathrm{T}}\right) \mathbf{w} \\ &= \left(\mathbf{S}_{\mathbf{w}}+\frac{\left(N_{1}+N_{2}\right) N_{1}-N_{1}^{2}}{N} \mathbf{m}_{1} \mathbf{m}_{1}^{\mathrm{T}}-\frac{N_{1} N_{2}}{N}\left(\mathbf{m}_{1} \mathbf{m}_{2}^{\mathrm{T}}+\mathbf{m}_{2} \mathbf{m}_{1}^{\mathrm{T}}\right) +\frac{\left(N_{1}+N_{2}\right) N_{2}-N_{2}^{2}}{N} \mathbf{m}_{2} \mathbf{m}_{2}^{\mathrm{T}}\right) \mathbf{w} \\ &= \left(\mathbf{S}_{\mathbf{w}}+\frac{N_{1} N_{2}}{N}\left(\mathbf{m}_{1} \mathbf{m}_{1}^{\mathrm{T}}-\mathbf{m}_{1} \mathbf{m}_{2}^{\mathrm{T}}-\mathbf{m}_{2} \mathbf{m}_{1}^{\mathrm{T}}+\mathbf{m}_{2} \mathbf{m}_{2}^{\mathrm{T}}\right)\right) \mathbf{w} \\ &= \left(\mathbf{S}_{\mathbf{w}}+\frac{N_{1} N_{2}}{N} \mathbf{S}_{\mathrm{B}}\right) \mathbf{w} \end{aligned}

となる。
以上から、(4.33)式が

\left(\mathbf{S}_{\mathbf{W}}+\frac{N_{1} N_{2}}{N} \mathbf{S}_{\mathbf{B}}\right) \mathbf{w}=N\left(\mathbf{m}_{1}-\mathbf{m}_{2}\right) \tag{4.37}

と書き直せることが示された。

演習 4.7

ロジスティックシグモイド関数

\sigma(a) = \frac{1}{1+ \exp(-a)} \tag{4.59}

\sigma(-a) = 1-\sigma(a)を満たすことを示せ.また,その逆関数が\sigma^{-1}(y) = \ln {y/(1-y)}で与えられることを示せ.


\begin{aligned} \sigma(-a) &= \frac{1}{1+\exp(a)} \\&= \frac{1}{1+\left\{ 1/\exp(-a) \right\}} \\&= \frac{\exp(-a)}{\exp(-a)+1} \\&= 1-\frac{1}{1+\exp(-a)} \\&= 1-\sigma(a) \end{aligned}

逆関数についてy=\sigma(a)としたとき

y=\frac{1}{1+\exp(-a)}
\begin{aligned} \exp(-a)&=\frac{1}{y}-1 \\&= \frac{1-y}{y} \end{aligned}
a=\ln(\frac{y}{1-y})

よって

\sigma^{-1}(y)=\ln\left(\frac{y}{1-y}\right)

演習 4.8

\begin{aligned} p\left(\mathcal{C}_{1} \mid \mathbf{x}\right) &=\frac{p\left(\mathbf{x} \mid \mathcal{C}_{1}\right) p\left(\mathcal{C}_{1}\right)}{p\left(\mathbf{x} \mid \mathcal{C}_{1}\right) p\left(\mathcal{C}_{1}\right)+p\left(\mathbf{x} \mid \mathcal{C}_{2}\right) p\left(\mathcal{C}_{2}\right)} \\ &=\frac{1}{1+\exp (-a)}=\sigma(a) \end{aligned} \tag{4.57}

a=\ln \frac{p\left(\mathbf{x} \mid \mathcal{C}_{1}\right) p\left(\mathcal{C}_{1}\right)}{p\left(\mathbf{x} \mid \mathcal{C}_{2}\right) p\left(\mathcal{C}_{2}\right)} \tag{4.58}

を使って,ガウス確率密度分布を用いた2クラス生成モデルにおけるクラスの事後確率に対する

p\left(\mathcal{C}_{1} \mid \mathbf{x}\right)=\sigma\left(\mathbf{w}^{\mathrm{T}} \mathbf{x}+w_{0}\right) \tag{4.65}

の結果を導出せよ.また,パラメータ\mathbf{w}w_0に対する結果

\mathbf{w} =\mathbf{\Sigma}^{-1}\left(\boldsymbol{\mu}_{1}-\boldsymbol{\mu}_{2}\right) \tag{4.66}

w_{0} =-\frac{1}{2} \boldsymbol{\mu}_{1}^{\mathrm{T}} \boldsymbol{\Sigma}^{-1} \boldsymbol{\mu}_{1}+\frac{1}{2} \boldsymbol{\mu}_{2}^{\mathrm{T}} \boldsymbol{\Sigma}^{-1} \boldsymbol{\mu}_{2}+\ln \frac{p\left(\mathcal{C}_{1}\right)}{p\left(\mathcal{C}_{2}\right)} \tag{4.67}

を検証せよ.


(4.65)式を(4.57), (4.58), それから

p\left(\mathbf{x} \mid \mathcal{C}_{k}\right)=\frac{1}{(2 \pi)^{D / 2}} \frac{1}{|\mathbf{\Sigma}|^{1 / 2}} \exp \left\{-\frac{1}{2}\left(\mathbf{x}-\boldsymbol{\mu}_{k}\right)^{\mathrm{T}} \mathbf{\Sigma}^{-1}\left(\mathbf{x}-\boldsymbol{\mu}_{k}\right)\right\} \tag{4.64}

を利用して変形していく。

\begin{aligned} p(\mathcal{C}_1\mid \mathbf{x}) &= \sigma\left(\ln \frac{p\left(\mathbf{x}\mid \mathcal{C}_{1}\right) p\left(\mathcal{C}_{1}\right)}{p\left(\mathbf{x} \mid \mathcal{C}_{2}\right) p\left(\mathcal{C}_{2}\right)}\right) \\ &=\sigma\left(\ln \frac{\exp \left\{ -\frac{1}{2} \left(\mathbf{x}-\boldsymbol{\mu}_{1}\right)^{\mathrm{T}} \mathbf{\Sigma}^{-1}\left(\mathbf{x}-\boldsymbol{\mu}_{1}\right)\right\}}{\exp \left\{ -\frac{1}{2} \left(\mathbf{x}-\boldsymbol{\mu}_{2}\right)^{\mathrm{T}} \mathbf{\Sigma}^{-1}\left(\mathbf{x}-\boldsymbol{\mu}_{2}\right)\right\}}+\ln \frac{p\left(\mathcal{C}_{1}\right)}{p\left(\mathcal{C}_{2}\right)}\right) \\ &=\sigma\left(-\frac{1}{2}\left\{\left(\mathbf{x}-\boldsymbol{\mu}_{1}\right)^{\mathrm{T}} \mathbf{\Sigma}^{-1}\left(\mathbf{x}-\boldsymbol{\mu}_{1}\right)\right\}+\frac{1}{2}\left\{\left(\mathbf{x}-\boldsymbol{\mu}_{2}\right)^{\mathrm{T}} \mathbf{\Sigma}^{-1}\left(\mathbf{x}-\boldsymbol{\mu}_{2}\right)\right\}+\ln \frac{p\left(\mathcal{C}_{1}\right)}{p\left(\mathcal{C}_{2}\right)}\right)\\ &=\sigma\left(\boldsymbol{\mu}_{1}^{\mathrm{T}} \mathbf{\Sigma}^{-1} \mathbf{x}-\frac{1}{2} \boldsymbol{\mu}_{1}^{\mathrm{T}} \mathbf{\Sigma}^{-1} \boldsymbol{\mu}_{1}-\boldsymbol{\mu}_{2}^{\mathrm{T}} \mathbf{\Sigma}^{-1} \mathbf{x}+\frac{1}{2} \boldsymbol{\mu}_{2}^{\mathrm{T}} \mathbf{\Sigma}^{-1} \boldsymbol{\mu}_{2}+\ln \frac{p\left(\mathcal{C}_{1}\right)}{p\left(\mathcal{C}_{2}\right)}\right)\\ &=\sigma\left(\left((\mathbf{\Sigma}^{-1})^{\mathrm{T}}\left(\boldsymbol{\mu}_{1}-\boldsymbol{\mu}_{2}\right)\right)^{\mathrm{T}} \mathbf{x}-\frac{1}{2} \boldsymbol{\mu}_{1}^{\mathrm{T}} \mathbf{\Sigma}^{-1} \boldsymbol{\mu}_{1}+\frac{1}{2} \boldsymbol{\mu}_{2}^{\mathrm{T}} \mathbf{\Sigma}^{-1} \boldsymbol{\mu}_{2}+\ln \frac{p\left(\mathcal{C}_{1}\right)}{p\left(\mathcal{C}_{2}\right)}\right) \end{aligned}

(\mathbf{\Sigma}^{-1})^{\mathrm{T}} = \mathbf{\Sigma}^{-1}なので、\mathbf{w} = \mathbf{\Sigma}^{-1}(\boldsymbol{\mu}_1 - \boldsymbol{\mu}_2) \quad (4.66)\displaystyle w_0 = -\frac{1}{2} \boldsymbol{\mu}_{1}^{\mathrm{T}} \mathbf{\Sigma}^{-1} \boldsymbol{\mu}_{1}+\frac{1}{2} \boldsymbol{\mu}_{2}^{\mathrm{T}} \mathbf{\Sigma}^{-1} \boldsymbol{\mu}_{2}+\ln \frac{p\left(\mathcal{C}_{1}\right)}{p\left(\mathcal{C}_{2}\right)} \quad (4.67)の定義を用いると,

p(\mathcal{C}_1\mid \mathbf{x}) = \sigma(\mathbf{w}^{\mathrm T}\mathbf{x}+w_0) \tag{4.65}

と書ける。

演習 4.9

クラスの事前確率p(\mathcal{C}_k)=\pi_kと一般的なクラスの条件付き確率密度p(\boldsymbol{\phi}\mid \mathcal{C}_k)によって定義されるKクラス分類問題の生成モデルを考える.ここで\boldsymbol{\phi}は入力特徴ベクトルである.学習データ集合\{ \boldsymbol{\phi}_n, \mathbf{t}_n \}が与えられたと仮定する.ここで,n=1,\ldots,Nであり,\mathbf{t}_nは,1-of-K符号化法を使う長さKの2値目的変数ベクトルである.つまり,パターンnのクラスが\mathcal{C}_kである場合,2値目的変数ベクトルは構成要素t_{nj} = I_{jk}を持つ.データがこのモデルから独立に抽出されると仮定すると,その事前確率に対する最尤解が以下の式で与えられることを示せ.

\pi_k = \frac{N_k}{N} \tag{4.159}

ここで,N_kはクラス\mathcal{C}_kに割り当てられるデータの個数である.


「パターンnのクラスが\mathcal{C}_kである場合,2値目的変数ベクトルは構成要素t_{nj} = I_{jk}を持つ」というのは、j=kであればt_{nk} = I_{kk} = 1, j\neq kであれば0ということである。

2クラス分類のときの尤度関数

p(\mathbf{t},\mathbf{X}\mid \pi, \boldsymbol{\mu}_1,\boldsymbol{\mu}_2,\mathbf{\Sigma}) = \prod_{n=1}^{N}[\pi \mathcal{N}(\mathbf{x}_n \mid \boldsymbol{\mu}_1,\mathbf{\Sigma})]^{t_n}[(1-\pi)\mathcal{N}(\mathbf{x}_n \mid \boldsymbol{\mu}_2,\mathbf{\Sigma})]^{1-t_n} \tag{4.71}

を多クラスに拡張したい。

\mathbf{I}_k=\begin{pmatrix} 0 \\ 0 \\ \vdots \\ 1 \\ \vdots \\ 0 \\ \end{pmatrix}

というk番目の要素が1で他が0であるベクトルを用いて、多クラスの尤度関数p(\{\mathbf{t}_n, \boldsymbol{\phi}_n \}\mid \{ \pi_k \} )は以下のようになる。(\mathbf{I}_k^\mathrm{T}\mathbf{t}_n = t_{nk}である。)

\begin{aligned} p(\{\mathbf{t}_n, \boldsymbol{\phi}_n \}\mid \{ \pi_k \} ) = \prod_{n=1}^{N}\prod_{k=1}^{K}[\pi_k p(\boldsymbol{\phi}_n\mid \mathcal{C}_k)]^{\mathbf{I}_k^\mathrm{T}\mathbf{t}_n} \end{aligned}

最大化するにあたって、これの対数尤度関数をとる。

\begin{aligned} \ln p(\{\mathbf{t}_n, \boldsymbol{\phi}_n \}\mid \{ \pi_k \} ) = \sum_{n=1}^{N}\sum_{k=1}^{K}\mathbf{I}_k ^\mathrm{T}\mathbf{t}_n (\ln\pi_k + p(\boldsymbol{\phi}_n\mid \mathcal{C}_k)) \end{aligned}

ここで、最大化に関係する部分だけ取り出し、\sum_{k=1}^{K} \pi_k = 1であることを考慮してラグランジュの未定乗数法を用いる(あまり本来の使用法と違う気もしますが使えるので)と

\begin{aligned} L = \sum_{n=1}^{N}\sum_{k=1}^{K}\mathbf{I}_k ^\mathrm{T}\mathbf{t}_n \ln\pi_k -\lambda \left\{ \left(\sum_{k=1}^{K}\pi_k \right) -1 \right\} \\ \end{aligned}

すべてのkにおける\pi_k\lambdaLを微分し、0を取ると

\begin{aligned} \frac{\partial L}{\partial \pi_k} &= \sum_{n=1}^{N}\frac{\mathbf{I}_k ^\mathrm{T} \mathbf{t}_n}{\pi_k}-\lambda = 0 \\ \frac{\partial L}{\partial \lambda} &= \left( \sum_{k=1}^{K}\pi_k \right) -1 = 0 \end{aligned}

となる。上式の分母を払うと

\lambda\pi_k = \sum_{n=1}^N \mathbf{I}_k ^\mathrm{T} \mathbf{t}_n

ここで\sum_{n=1}^N \mathbf{I}_k ^\mathrm{T} \mathbf{t}_nは定義からクラス\mathcal{C}_kに割り当てられるデータの個数N_kであるから、\pi_k = N_k/\lambdaとなる。

一方、すべてのkについて足し合わせると

\begin{aligned} \sum_{k=1}^K \lambda\pi_k &= \sum_{k=1}^K \sum_{n=1}^N \mathbf{I}_k ^\mathrm{T} \mathbf{t}_n \\ \lambda \left( \sum_{k=1}^K \pi_k \right) &= \sum_{k=1}^K N_k \\ \lambda &= N\quad \left(\because \frac{\partial L}{\partial \lambda} = 0から \left( \sum_{k=1}^{K}\pi_k \right) = 1 \right) \end{aligned}

より

\pi_k = \frac{N_k}{N}

となり、示された。

演習 4.10

演習問題4.9の分類モデルを考え,クラスの条件付き確率密度が共通の共分散行列を持つガウス分布によって与えられる,つまり,以下の式が成立すると仮定する.

p\left(\boldsymbol{\phi} \mid \mathcal{C}_{k}\right)=\mathcal{N}\left(\boldsymbol{\phi} \mid \boldsymbol{\mu}_{k}, \mathbf{\Sigma} \right) \tag{4.160}

クラス\mathcal{C}_kのガウス分布の平均に対する最尤解が以下の式で与えられることを示せ.

\boldsymbol{\mu}_k=\frac{1}{N_{k}}\sum_{n=1}^N t_{nk}\boldsymbol{\phi}_n \tag{4.161}

これは,クラス\mathcal{C}_kに割り当てられる特徴ベクトルの平均を表す.同様に,共通の共分散に対する最尤解が,以下の式で与えられることを示せ.

\mathbf{\Sigma}=\sum_{k=1}^{K} \frac{N_{k}}{N} \mathbf{S}_{k} \tag{4.162}

ここで

\mathbf{S}_{k}=\frac{1}{N_{k}} \sum_{n=1}^{N} t_{nk}\left(\boldsymbol{\phi}_{n}-\boldsymbol{\mu}_{k}\right)\left(\boldsymbol{\phi}_{n}-\boldsymbol{\mu}_{k}\right)^{\mathrm{T}} \tag{4.163}

である.よって,\mathbf{\Sigma}は,各クラスのデータの共分散の重み付き平均で与えられる.重み付け係数はクラスの事前確率で与えられる.


※ 演習問題4.9でのp(\boldsymbol{\phi}_n\mid \mathcal{C}_k)に正規分布\displaystyle \mathcal{N}(\boldsymbol{\phi}_n \mid \boldsymbol{\mu}_k,\mathbf{\Sigma}) = \frac{1}{(2\pi)^{D/2}|\mathbf{\Sigma}|^{1/2}} \exp\left\{ -\frac{1}{2}(\boldsymbol{\phi}_n - \boldsymbol{\mu}_k)^{\mathrm T}\mathbf{\Sigma}^{-1}(\boldsymbol{\phi}_n - \boldsymbol{\mu}_k) \right\}を仮定した場合になる。

演習問題4.9を利用すると尤度関数は

p\left(\left\{\mathbf{t}_{n}, \boldsymbol{\phi}_{n}\right\} \mid\left\{\pi_{k}\right\}\right)=\prod_{n=1}^{N} \prod_{k=1}^{K}\left[\pi_{k} \mathcal{N}(\boldsymbol{\phi}_n \mid \boldsymbol{\mu}_k,\mathbf{\Sigma})p(\mathcal{C}_k)\right]^{t_{nk}}

であり、対数尤度関数は\boldsymbol{\mu}_k\mathbf{\Sigma}に依存する項を抜き出して残りを\textrm{const.}とすると

\begin{aligned} \ln p\left(\left\{\mathbf{t}_{n}, \boldsymbol{\phi}_{n}\right\} \mid\left\{\pi_{k}\right\}\right) &=\sum_{n=1}^{N} \sum_{k=1}^{K} t_{nk}\left( \ln \pi_k +\ln \mathcal{N}(\boldsymbol{\phi}_n \mid \boldsymbol{\mu}_k,\mathbf{\Sigma})+\ln p(\mathcal{C}_k)\right) \\ &=-\frac{1}{2}\sum_{n=1}^{N} \sum_{k=1}^{K} t_{nk}\left( \ln |\mathbf{\Sigma}| + (\boldsymbol{\phi}_n - \boldsymbol{\mu}_k)^{\mathrm T}\mathbf{\Sigma}^{-1}(\boldsymbol{\phi}_n - \boldsymbol{\mu}_k) \right) + \textrm{const.} \end{aligned}

\boldsymbol{\mu}_kについての微分を0とすると(※ここでkは特定の値をとる)

\begin{aligned} \frac{\partial}{\partial \boldsymbol{\mu}_{k}} \ln p &=-\frac{1}{2} \sum_{n=1}^{N}\left\{t_{n k} \frac{\partial}{\partial \boldsymbol{\mu}_{k}}\left(\boldsymbol{\phi}_{n}-\boldsymbol{\mu}_{k}\right)^{\mathrm{T}} \mathbf{\Sigma}^{-1}\left(\boldsymbol{\phi}_{n}-\boldsymbol{\mu}_{k}\right)\right\} \\ &=\sum_{n=1}^{N}\left\{t_{n k} \mathbf{\Sigma}^{-1}\left(\boldsymbol{\phi}_{n}-\boldsymbol{\mu}_{k}\right)\right\} \\ &=\mathbf{\Sigma}^{-1} \sum_{n=1}^{N}\left\{t_{n k}\left(\boldsymbol{\phi}_{n}-\boldsymbol{\mu}_{k}\right)\right\} \\ &=\mathbf{\Sigma}^{-1}\left(\sum_{n=1}^{N} t_{n k} \boldsymbol{\phi}_{n}-\sum_{n=1}^{N} t_{n k} \boldsymbol{\mu}_{k}\right) \\ &=\mathbf{\Sigma}^{-1}\left(\sum_{n=1}^{N} t_{n k} \boldsymbol{\phi}_{n}-N_{k} \boldsymbol{\mu}_{k}\right) = 0 \end{aligned}

以上から

\boldsymbol{\mu}_{k}=\frac{1}{N_{k}} \sum_{n=1}^{N} t_{n k} \boldsymbol{\phi}_{n} \tag{4.161}

が求まる。

続いて\mathbf{S}_kをうまく使えるように対数尤度関数を変形すると、

\begin{aligned} \ln p &=\sum_{n=1}^{N} \sum_{k=1}^{K}\left\{t_{n k} \ln \mathcal{N}\left(\boldsymbol{\phi}_{n} \mid \boldsymbol{\mu}_{k}, \mathbf{\Sigma}\right)\right\} \\ &=\sum_{n=1}^{N} \sum_{k=1}^{K}\left\{t_{n k}\left[-\frac{1}{2} \ln |\mathbf{\Sigma}|-\frac{1}{2}\left(\boldsymbol{\phi}_{n}-\boldsymbol{\mu}_{k}\right)^{\mathrm{T}} \mathbf{\Sigma}^{-1}\left(\boldsymbol{\phi}_{n}-\boldsymbol{\mu}_{k}\right)\right]\right\}+\mathrm{const.} \\ &=-\frac{1}{2} \sum_{n=1}^{N} \sum_{k=1}^{K} t_{n k} \ln |\mathbf{\Sigma}|-\frac{1}{2} \sum_{n=1}^{N} \sum_{k=1}^{K} t_{n k}\left(\boldsymbol{\phi}_{n}-\boldsymbol{\mu}_{k}\right)^{\mathrm{T}} \mathbf{\Sigma}^{-1}\left(\boldsymbol{\phi}_{n}-\boldsymbol{\mu}_{k}\right)+\mathrm{const.} \\ &=-\frac{N}{2} \ln |\mathbf{\Sigma}|-\frac{1}{2} \sum_{n=1}^{N} \sum_{k=1}^{K} t_{n k}\left(\boldsymbol{\phi}_{n}-\boldsymbol{\mu}_{k}\right)^{\mathrm{T}} \mathbf{\Sigma}^{-1}\left(\boldsymbol{\phi}_{n}-\boldsymbol{\mu}_{k}\right)+\mathrm{const.} \\ &=-\frac{N}{2} \ln |\mathbf{\Sigma}|-\frac{1}{2} \sum_{n=1}^{N} \sum_{k=1}^{K} t_{n k} \operatorname{Tr}\left[\mathbf{\Sigma}^{-1}\left(\boldsymbol{\phi}_{n}-\boldsymbol{\mu}_{k}\right)\left(\boldsymbol{\phi}_{n}-\boldsymbol{\mu}_{k}\right)^{\mathrm{T}}\right]+\mathrm{const.} \\ &=-\frac{N}{2} \ln |\mathbf{\Sigma}|-\frac{1}{2} \sum_{k=1}^{K}\left( N_k \sum_{n=1}^{N} \frac{1}{N_k} t_{nk} \operatorname{Tr}\left[\mathbf{\Sigma}^{-1}\left(\boldsymbol{\phi}_{n}-\boldsymbol{\mu}_{k}\right)\left(\boldsymbol{\phi}_{n}-\boldsymbol{\mu}_{k}\right)^{\mathrm{T}}\right]\right) +\mathrm{const.} \\ &=-\frac{N}{2} \ln |\mathbf{\Sigma}|-\frac{1}{2} \sum_{k=1}^{K} N_k \left( \operatorname{Tr}\left[ \mathbf{\Sigma}^{-1} \frac{1}{N_k} \sum_{n=1}^{N} t_{nk} \left(\boldsymbol{\phi}_{n}-\boldsymbol{\mu}_{k}\right)\left(\boldsymbol{\phi}_{n}-\boldsymbol{\mu}_{k}\right)^{\mathrm{T}}\right]\right) +\mathrm{const.} \\ &=-\frac{N}{2} \ln |\mathbf{\Sigma}|-\frac{1}{2} \sum_{k=1}^K N_k \operatorname{Tr}\left[\mathbf{\Sigma}^{-1} \mathbf{S}_k\right]+\mathrm{const.} \end{aligned}

\mathbf{\Sigma}についての微分をとって0とする。演習問題2.34と同様に変形していくと、

\begin{aligned} \frac{\partial}{\partial \mathbf{\Sigma}}\ln p &= -\frac{N}{2}(\mathbf{\Sigma}^{-1})^{\mathrm T} - \frac{1}{2}\sum_{k=1}^K N_k \frac{\partial}{\partial \mathbf{\Sigma}}\operatorname{Tr}\left[\mathbf{\Sigma}^{-1} \mathbf{S}_k\right] \\ &= -\frac{N}{2}(\mathbf{\Sigma}^{-1})^{\mathrm T} + \frac{1}{2}\sum_{k=1}^K N_k (\mathbf{\Sigma}^{-1} \mathbf{S}_k \mathbf{\Sigma}^{-1})^{\mathrm T} = 0 \end{aligned}

転置をとって移項すると

\mathbf{\Sigma}^{-1} = \sum_{k=1}^K \frac{N_k}{N} \mathbf{\Sigma}^{-1} \mathbf{S}_k \mathbf{\Sigma}^{-1}

左と右からそれぞれ\mathbf{\Sigma}をかければ

\mathbf{\Sigma} = \sum_{k=1}^K \frac{N_k}{N}\mathbf{S}_k \tag{4.162}

を得る。

演習 4.11

各々がLの離散状態を取ることのできるM個の要素を持つ特徴ベクトル\boldsymbol{\phi}に対するKクラスの分類問題を考える.1-of-L符号化法によって成分の値は表現されるとする.さらに,クラス\mathcal{C}_kに対し\boldsymbol{\phi}M個の成分が独立であり,クラスの条件付き確率密度は特徴ベクトルの要素に分解できると仮定する.クラスの事後確率を記述しているソフトマックス関数の引数に現れる

a_{k}=\ln \left(p\left(\mathbf{x} \mid \mathcal{C}_{k}\right) p\left(\mathcal{C}_{k}\right)\right) \tag{4.63}

によって与えられる量a_k\boldsymbol{\phi}の成分の線形関数であることを示せ.これが,8.2.2節で議論されるナイーブベイズモデルの一例であることに注意せよ.


※ P.200 4.2.3 離散特徴の議論で行われていたのは、1つの特徴が1次元x_{i}かつ離散値が\{0, 1\}の場合であり、この入力がD個ある場合である。この問題では一般的に各離散変数がL個の値を取りうるM次元ベクトルで表されるとする。

仮定から「各々がLの離散状態を取ることのできるM個の要素を持つ特徴ベクトル\boldsymbol{\phi}」とあるので、要素がL個のベクトル\mathbf{x}_{m}M個並んだ以下のようなM \times L行列の\boldsymbol{\phi}を考える(なので\boldsymbol{\phi}はベクトルではなく行列であるはず)。

\boldsymbol{\phi}=\begin{pmatrix} \mathbf{x}_1^{\mathrm{T}} \\ \mathbf{x}_2^{\mathrm{T}} \\ \vdots \\ \mathbf{x}_M^{\mathrm{T}} \\ \end{pmatrix}

この\mathbf{x}_{m}L次元ベクトルで、1-of-L符号化法によって成分の値は表現されると仮定されているので、いずれか1つの要素が1で、他は0である。

今、仮定から「クラス\mathcal{C}_kに対し\boldsymbol{\phi}M個の成分(\boldsymbol{\phi}_{m})が独立」とあり、これは8.2条件付き独立性の概念を用いると(※上巻ではまだこの説明が明確にされていないが)、p(\boldsymbol{\phi}_{1}, \boldsymbol{\phi}_{2}, \ldots, \boldsymbol{\phi}_{M} \mid \mathcal{C}_{k}) = \prod_{m=1}^{M} p(\boldsymbol{\phi}_{m} \mid \mathcal{C}_{k})、すなわち

\begin{aligned} p(\boldsymbol{\phi} \mid \mathcal{C}_k) = \prod_{m=1}^{M} p(\boldsymbol{\phi}_{m} \mid \mathcal{C}_{k}) \end{aligned}

と書けるうえ、「クラスの条件付き確率密度は特徴ベクトルの要素に分解できる」と仮定するならば、多項分布を用いて(※離散値が二値変数だったときは(4.81)式がベルヌーイ分布の形だったが、1-of-L符号化を行う多値変数の場合は多項分布になる。2.2 多値変数を参照)、クラスの条件付き確率分布は

p(\boldsymbol{\phi}_{m} \mid \mathcal{C}_{k}) = \prod_{l=1}^{L}\mu_{kml}^{\phi_{ml}}

ここで、\mu_{k m l}は、あるクラス\mathcal{C}_{k}において\boldsymbol{\phi}_{m} = \mathbf{x}_{m}^{\mathrm{T}}l番目の成分を表す\phi_{m l}について\phi_{m l} = 1となる確率を表している。

さて、「生成的アプローチでは,モデル化されたクラスの条件付き確率密度p(\mathbf{x} \mid \mathcal{C}_{k})とクラスの事前確率p(\mathcal{C}_{k})からベイズ定理を使って,事後確率p(\mathcal{C}_{k}\mid\mathbf{x})を計算する」(P.195)もので(※この問題では\mathbf{x}\boldsymbol{\phi}に置き換えて考える)、多クラスの場合の(4.63)式は

\begin{aligned} a_k & =\ln p\left(\boldsymbol{\phi} \mid \mathcal{C}_k\right) p\left(\mathcal{C}_k\right) \\ & =\ln p\left(\mathcal{C}_k\right)+\sum_{m=1}^M \ln p\left(\boldsymbol{\phi}_m \mid \mathcal{C}_k\right) \\ & =\ln p\left(\mathcal{C}_k\right)+\sum_{m=1}^M \sum_{l=1}^L \phi_{m l} \ln \mu_{k m l} \end{aligned}

となる。これはa_k\boldsymbol{\phi}の成分\phi_{ml}について線形であることを示している。

(PRML下巻 P.93 8.2.2節より)クラス分類問題のための1手法である ナイーブベイズ (naive Bayes) モデルと呼ばれる方法でも、有向分離に関連するグラフ構造が見られる。ナイーブベイズモデルでは条件付き独立性によってモデル構造が単純化される。観測変数がD次元ベクトル \mathrm{x}=\left(x_1, \ldots, x_D\right)^{\mathrm{T}} から成るとし,観測値 \mathrm{x}K 個のクラスのうちの1つに対応させたいとする、1-of-K 符号化法を用いれば,クラスを K 次元 2 値ベクトルで表せる。さらに,クラスラベル上の多項事前分布 p(\mathbf{z} \mid \boldsymbol{\mu}) を導入し \left(\boldsymbol{\mu}\right.k 番目の成分 \mu_k をクラス\mathcal{C}_k の事前確率とする),これと観測値べクトル \mathbf{x} の条件付き分布 p(\mathbf{x} \mid \mathbf{z}) とを用いれば,生成モデルを定義できる.ナイーブベイズモデルにおける重要な仮定は,クラス\mathbf{z}で条件付けられたとき,入力変数x_1, \ldots, x_Dの分布が互いに独立であるということである.

演習 4.12

\sigma(a) = \frac{1}{1+ \exp(-a)} \tag{4.59}

で定義されるロジスティックシグモイド関数の微分に対する関係

\frac{d\sigma}{da} = \sigma(1-\sigma) \tag{4.88}

を検証せよ.


\exp(-a)=e^{-a}である。

\begin{aligned} \frac{ \mathrm{d} }{ \mathrm{d} a} \sigma(a) &= \frac{ \mathrm{d} }{ \mathrm{d} a} \frac{1}{1+ \exp(-a)} \\ &= - \exp(-a) \frac{-1}{\left(1+ \exp(-a)\right)^2} \\ &= \frac{1}{1+ \exp(-a)} \frac{\exp(-a)}{1+ \exp(-a)} \\ &= \frac{1}{1+ \exp(-a)} \frac{(1+\exp(-a))-1}{1+ \exp(-a)} \\ &= \frac{1}{1+ \exp(-a)} \left(1- \frac{1}{1+ \exp(-a)}\right) \\ &= \sigma(1-\sigma) \end{aligned}

演習 4.13

ロジスティックシグモイドの微分に対する結果

\frac{d\sigma}{da} = \sigma(1-\sigma) \tag{4.88}

を使って,ロジスティック回帰モデルに対する誤差関数

E(\mathbf{w})=-\ln p(\mathsf{t} \mid \mathbf{w})=-\sum_{n=1}^{N}\left\{t_{n} \ln y_{n}+\left(1-t_{n}\right) \ln \left(1-y_{n}\right)\right\} \tag{4.90}

の微分が,

\nabla E(\mathbf{w})=\sum_{n=1}^{N}\left(y_{n}-t_{n}\right) \phi_{n} \tag{4.91}

で与えられることを示せ.


Ey_nの関数、y_na_nの関数、a_n\mathbf{w}の関数なので、連鎖律を利用する。

すなわち\nabla E(\mathbf{w}) = \frac{\partial E}{\partial y_n}\frac{\partial y_n}{\partial a_n}\nabla a_n としてそれぞれの項を求めれば良い。

E(\mathbf{w})=-\sum_{n=1}^{N}\left\{t_{n} \ln y_{n}+\left(1-t_{n}\right) \ln \left(1-y_{n}\right)\right\} より

\begin{aligned} \frac{\partial E}{\partial y_n} &=-\sum_{n=1}^{N}\left( \frac{t_{n}}{y_{n}}-\frac{1-t_{n}}{1-y_{n}} \right)\\ &=-\sum_{n=1}^{N}\left(\frac{t_{n}\left(1-y_{n}\right)-y_{n}\left(1-t_{n}\right)}{y_{n}\left(1-y_{n}\right)}\right) \\ &=\sum_{n=1}^{N} \frac{y_{n}-t_{n}}{y_{n}\left(1-y_{n}\right)} \end{aligned} \tag{1}

y_n=\sigma(a_n) より \frac{d\sigma}{da} = \sigma(1-\sigma)を利用して

\frac{\partial y_{n}}{\partial a_{n}}=\frac{\partial \sigma\left(a_{n}\right)}{\partial a_{n}}=\sigma\left(a_{n}\right)\left(1-\sigma\left(a_{n}\right)\right)=y_{n}\left(1-y_{n}\right) \tag{2}

a_n = \mathbf{w}^{\mathrm T}\boldsymbol{\phi}_nより

\nabla a_n = \boldsymbol{\phi}_n \tag{3}

よって(1), (2), (3)より

\nabla E(\mathbf{w}) = \frac{\partial E}{\partial y_n}\frac{\partial y_n}{\partial a_n}\nabla a_n = \sum_{n=1}^{N}(y_n - t_n)\boldsymbol{\phi}_n

となり、(4.91)式が示された。

演習 4.14

線形分離可能なデータ集合に対し,ロジスティック回帰モデルの最尤解が,クラスを分離する決定境界\mathbf{w}^{\mathrm{T}}\boldsymbol{\phi}(\mathbf{x})=0を満足するベクトル\mathbf{w}に対し,その値を\inftyとすることで得られることを示せ.


データ集合\{ \boldsymbol{\phi}_n, t_n\}, t_n \in \{0, 1\}について、もしデータ集合が線形分離可能ならば、2つのクラスを分ける決定境界の超平面は\mathbf{w}^{\mathrm{T}} \boldsymbol{\phi}(\mathbf{x})=0で与えられ、

\mathbf{w}^{\mathrm{T}} \boldsymbol{\phi}_{n} \left\{\begin{aligned} \geq 0 & \text { if } t_{n}=1 \\ <0 & \text { otherwise } (t_n = 0) \end{aligned}\right.

となる。

一方、最尤法で尤度の負の対数をとって誤差関数(4.90)を定義すると

E(\mathbf{w})=-\ln p(\mathsf{t} \mid \mathbf{w})=-\sum_{n=1}^{N}\left\{t_{n} \ln y_{n}+\left(1-t_{n}\right) \ln \left(1-y_{n}\right)\right\} \tag{4.90}

この勾配は

\nabla E(\mathbf{w})=\sum_{n=1}^{N}\left(y_{n}-t_{n}\right) \phi_{n} \tag{4.91}

すなわち、すべてのnについてy_n = \sigma(\mathbf{w}^{\mathrm T}\boldsymbol{\phi}_n)=t_nのときに最小化される。
ここで、t_n \in \{ 0, 1\}なのでy_n \in \{ 0, 1\}になるが、これはシグモイド関数の両端、つまり\mathbf{w}^{\mathrm T}\boldsymbol{\phi}_n = \pm \inftyのときに相当する。\boldsymbol{\phi}_nは固定なので、\mathbf{w}の大きさが\inftyの時に相当する。このときE(\mathbf{w})は最小化されロジスティック回帰モデルの最尤解が得られることになる。

演習 4.15

\mathbf{H}=\nabla \nabla E(\mathbf{w})=\sum_{n=1}^{N} y_{n}\left(1-y_{n}\right) \boldsymbol{\phi}_{n} \boldsymbol{\phi}_{n}^{\mathrm{T}}=\mathbf{\Phi}^{\mathrm{T}} \mathbf{R} \mathbf{\Phi} \tag{4.97}

で与えられるロジスティック回帰モデルのヘッセ行列\mathbf{H}が正定値行列であることを示せ.ここで\mathbf{R}は,要素をy_n(1-y_n)とする対角行列であり,y_nは,入力ベクトル\mathbf{x}_nに対するロジスティック回帰モデルの出力である.したがって,誤差関数は\mathbf{w}の凸関数であり,唯一の最小解を持つことを示せ.


P.206の設定から\mathbf{\Phi}N\times Mの行列であり、\mathbf{H} = \mathbf{\Phi}^{\mathrm{T}}\mathbf{\Phi}M \times M行列、\mathbf{R}は要素をy_n(1-y_n)とするN\times Nの対角行列であり、0 \lt y_{n} \lt 1から要素はすべて正である。ある対角行列について、対角要素がすべて正であるときかつそのときに限って正定値行列であるから、対角行列\mathbf{R}は正定値行列である(※これは定義からすぐに証明できる)。さらに、\mathbf{R}^{1/2} = \operatorname{diag}(\sqrt{y_{1}(1-y_{1})},\ldots, \sqrt{y_{n}(1-y_{n})})として、\mathbf{R} = \operatorname{diag}(y_{1}(1-y_{1}),\ldots ,y_{n}(1-y_{n})) = \mathbf{R}^{1/2}\mathbf{R}^{1/2}が成立する。

ヘッセ行列\mathbf{H}が正定値行列であることを示すためには、正定値行列の定義から、任意の零ベクトルでない\mathbf{x}に対して\mathbf{x}^\mathrm{T} \mathbf{Hx} = \mathbf{x}^\mathrm{T} \mathbf{\Phi}^\mathrm{T} \mathbf{R} \mathbf{\Phi} \mathbf{x}>0が成り立つことを確認すれば良い。左辺から変形していくと

\begin{aligned} \mathbf{x}^\mathrm{T} \mathbf{\Phi}^\mathrm{T} \mathbf{R} \mathbf{\Phi} \mathbf{x} &= \left(\mathbf{x^{\mathrm{T}}\Phi^{\mathrm{T}}} (\mathbf{R}^{1/2})^{\mathrm{T}}\right)\mathbf{R}^{1/2}\mathbf{\Phi x} \\ &= \| \mathbf{R}^{1/2}\mathbf{\Phi x}\| ^{2} \gt 0 \end{aligned}

となり、常に正であることが示される。以上よりヘッセ行列\mathbf{H}が正定値行列であることが示された。

また、最小値\mathbf{w}^{\star}周りでの誤差関数E(\mathbf{w})のテイラー展開

E(\mathbf{w})=E\left(\mathbf{w}^{\star}\right)+\frac{1}{2}\left(\mathbf{w}-\mathbf{w}^{\star}\right)^{\mathrm{T}} \mathbf{H}\left(\mathbf{w}-\mathbf{w}^{\star}\right)

を考える。ここで\mathbf{w} = \mathbf{w}^{\star} + \lambda \mathbf{v}となるような任意の零ベクトルでない\mathbf{v}を用いて最小値からのずれを表現すると

\frac{\partial^2 E}{\partial \lambda^2}=\mathbf{v}^{\mathrm{T}} \mathbf{H} \mathbf{v} \gt 0

であるから、E(\mathbf{w})は下に凸(凸関数)である。またE(\mathbf{w})の最小値において

\mathbf{H}\left(\mathbf{w}-\mathbf{w}^{\star}\right)=0

となるが、\mathbf{H}は正定値行列であり逆行列\mathbf{H}^{-1}が必ず存在するのでこれが成立するためには\mathbf{w} = \mathbf{w}^{\star}、すなわち最小値は唯一であることが示される。

演習 4.16

t=0またはt=1に対応する2クラスの1つに属することが知られている各観測値\mathbf{x}_nにおける2値分類問題を考える.このとき,学習データがときどき間違ったラベルを付けられるため,学習データの収集手順は完全なものではないと仮定する.すべてのデータ\mathbf{x}_nに対し,クラスラベルの値t_nを与える代わりに,t_n = 1となる確率を表現する値\pi_nを与える.確率モデルp(t=1\mid \boldsymbol{\phi})が与えられた場合そのようなデータ集合に適切な対数尤度関数を記述せよ.


※尤度関数の解釈を問う問題?

もし完全にラベル付けt_nが間違っていない場合、p_n = p(t_n = 1 \mid \boldsymbol{\phi}(\mathbf{x}_n))とすると、ある観測点\mathbf{x}_nについての尤度関数は

p(t_n\mid \boldsymbol{\phi}(\mathbf{x}_n)) = p_n^{t_n}(1-p_n)^{1-t_n}

すべてのデータ\{\mathsf{t}, \boldsymbol{\phi}(\mathbf{x}_n)\}に対する尤度関数と対数尤度関数は

p(\mathsf{t}\mid\boldsymbol{\phi}(\mathbf{x}_n)) = \prod_{n=1}^{N}p_n^{t_n}(1-p_n)^{1-t_n}
\ln p(\mathsf{t}\mid\boldsymbol{\phi}(\mathbf{x}_n)) = \sum_{n=1}^{N}\{t_n \ln p_n + (1-t_n)\ln(1-p_n)\}

となる。この式を考えると、t_n=1のときに対数尤度関数を\ln p_nだけ増加させ、t_n=0のときには\ln(1-p_n)だけ増加させると解釈することができる。この解釈に基づくと、確率\pi_nt_n=1となる(確率1-\pi_nt_n=0となる)とき、対数尤度関数は

\begin{aligned} \ln p(\mathsf{t}\mid\boldsymbol{\phi}(\mathbf{x}_n)) &= \sum_{n=1}^{N}\{\pi_n \ln p_n + (1-\pi_n)\ln(1-p_n)\} \\ &=\sum_{n=1}^{N}\{\pi_n \ln p(t_n = 1 \mid \boldsymbol{\phi}(\mathbf{x}_n)) + (1-\pi_n)\ln(1-p(t_n = 1 \mid \boldsymbol{\phi}(\mathbf{x}_n)))\} \end{aligned}

で与えられることになる。

演習 4.17

ソフトマックス活性化関数

p\left(\mathcal{C}_{k} \mid \boldsymbol{\phi}\right)=y_{k}(\boldsymbol{\phi})=\frac{\exp \left(a_{k}\right)}{\sum_{j} \exp \left(a_{j}\right)} \tag{4.104}

の微分が,

\frac{\partial y_{k}}{\partial a_{j}}=y_{k}\left(I_{k j}-y_{j}\right) \tag{4.106}

によって与えられることを示せ.I_{kj}は単位行列の要素である。ここで,a_k

a_k = \mathbf{w}_k^{\mathrm T}\boldsymbol{\phi} \tag{4.105}

によって定義される.


※4.3.4 多クラスロジスティック回帰を参照。k=jk\neq jのときに分けて考える。

(i) k\neq jのとき

\begin{aligned} \frac{\partial y_k}{\partial a_j} &= \frac{\partial}{\partial a_j}\left( \frac{e^{a_k}}{\sum_j \exp(a_j)} \right) \\ &= -\frac{e^{a_k}e^{a_j}}{\left( \sum_j \exp(a_j) \right)^2} \\ &=-y_k y_j \end{aligned}

(ii) k = jのとき

\begin{aligned} \frac{\partial y_j}{\partial a_j} &= \frac{\partial}{\partial a_j}\left( \frac{e^{a_j}}{\sum_j \exp(a_j)} \right) \\ &= \frac{e^{a_j}}{\sum_j \exp(a_j)} - \frac{e^{a_j}\cdot e^{a_j}}{\left(\sum_j \exp(a_j) \right)^2} \\ &=y_j(1-y_j) \end{aligned}

これらをまとめると

\frac{\partial y_k}{\partial a_j} = y_k(I_{kj}-y_j) \tag{4.106}

となる。ここで、I_{kj}は単位行列のkj成分である。

演習 4.18

ソフトマックス活性化関数の微分の結果

\frac{\partial y_{k}}{\partial a_{j}}=y_{k}\left(I_{k j}-y_{j}\right) \tag{4.106}

を使って,交差エントロピー誤差

E\left(\mathbf{w}_{1}, \ldots, \mathbf{w}_{K}\right)=-\ln p\left(\mathbf{T} \mid \mathbf{w}_{1}, \ldots, \mathbf{w}_{K}\right)=-\sum_{n=1}^{N} \sum_{k=1}^{K} t_{n k} \ln y_{n k} \tag{4.108}

の勾配が

\nabla_{\mathbf{w}_{j}} E\left(\mathbf{w}_{1}, \ldots, \mathbf{w}_{K}\right)=\sum_{n=1}^{N}\left(y_{n j}-t_{n j}\right) \boldsymbol{\phi}_{n} \tag{4.109}

で与えられることを示せ.


※演習問題4.17を利用する。

まず前提として行列\mathbf{T}は、「クラス\mathcal{C}_kに属する特徴ベクトル\boldsymbol{\phi}_nに対する目的変数ベクトル\mathbf{t}_nが、k番目の要素で1で残りはすべて0となる1-of-K符号化法」で記述されるような行列(教科書P.208)である。

\nabla_{\mathbf{w}_j}Eについて微分のchain ruleを使う。
Eがすでにy_{nk}(=y_k(\boldsymbol{\phi}_n))の関数となっていることを用いると

\begin{aligned} \nabla_{\mathbf{w}_{j}} E(\mathbf{w}_1,\ldots,\mathbf{w}_K) &=\frac{\partial E}{\partial y_{nk}}\frac{\partial y_{nk}}{\partial a_j}\nabla_{\mathbf{w}_{j}} a_j \\ &=-\sum_{n=1}^{N} \sum_{k=1}^{K} \frac{t_{nk}}{y_{nk}}y_{nk}(I_{kj}-y_{nj})\boldsymbol{\phi}_n \\ &=-\sum_{n=1}^{N} \sum_{k=1}^{K} t_{nk}(I_{kj}-y_{nj})\boldsymbol{\phi}_n \\ &=\sum_{n=1}^{N} \left\{ y_{nj}\left( \sum_{k=1}^{K} t_{nk} \right) - \left( \sum_{k=1}^{K} t_{nk}I_{kj} \right) \right\}\boldsymbol{\phi}_n \end{aligned}

ここで、「前提」より\sum_{k=1}^{K}t_{nk}=1となり、k=jの場合I_{kj}=1で、k\neq jI_{kj}=0であることを用いると

\begin{aligned} \nabla E_{\mathbf{w}_{j}}(\mathbf{w}_1,\ldots,\mathbf{w}_K) &=\sum_{n=1}^{N} \left\{ y_{nj}\left( \sum_{k=1}^{K} t_{nk} \right) - \left( \sum_{k=1}^{K} t_{nk}I_{kj} \right) \right\}\boldsymbol{\phi}_n \\ &=\sum_{n=1}^{N} (y_{nj}-t_{nj})\boldsymbol{\phi}_n \end{aligned}

となり、(4.109)式が与えられた。

演習 4.19

4.3.5節で定義したプロビット回帰モデルに対し対数尤度の勾配および,対応するヘッセ行列を求めよ.これらは,IRLSを使って,プロビット回帰モデルのようなモデルを学習するために必要とされる量である.


※まずプロビット回帰モデルの定義と尤度関数を確認する。微分計算は結構大変で、演習問題4.13をうまく利用して進めていく。ヘッセ行列は勾配をもう一度偏微分することで求まる。

プロビット回帰モデルは

p(t_n=1\mid a_n) = y_n = \Phi(a_n) = \int_{-\infty}^{a_n}\mathcal{N}(\theta\mid 0,1)d\theta

で表現するモデルである。ここで、a_n = \mathbf{w}^{\mathrm T}\boldsymbol{\phi_n}である。

尤度関数は

p(\mathsf{t}\mid \mathbf{w}) = \prod_{n=1}^{N}y_n^{t_n}(1-y_n)^{1-t_n}

なので負の対数尤度(cross entropy誤差関数)は(4.90)と同形で

E(\mathbf{w}) = -\sum_{n=1}^N \{ t_n\ln y_n + (1-t_n)\ln(1-y_n) \}

を与える。

演習問題4.13と同様にして

\begin{aligned} \frac{\partial E(\mathbf{w})}{\partial y_{n}} &=\sum_{n=1}^{N} \frac{y_{n}-t_{n}}{y_{n}\left(1-y_{n}\right)} \\ \nabla_{\mathbf{w}} a_n &= \boldsymbol{\phi}_n \end{aligned}

であり、

\frac{\partial \Phi(a_n)}{\partial a_n}=\frac{\partial}{\partial a_n} \int_{-\infty}^{a_n} \mathcal{N}(\theta \mid 0,1) \mathrm{d} \theta=\mathcal{N}(a_n \mid 0,1)

となることから\nabla_{\mathbf{w}} E(\mathbf{w})を求めると

\begin{aligned} \nabla_{\mathbf{w}} E(\mathbf{w})&=\sum_{n=1}^{N} \frac{\partial E_{n}}{\partial y_{n}} \frac{\partial y_{n}}{\partial a_{n}} \nabla_{\mathbf{w}} a_{n} \\ &=\sum_{n=1}^{N} \frac{y_{n}-t_{n}}{y_{n}\left(1-y_{n}\right)} \mathcal{N}\left(a_{n} \mid 0,1\right) \boldsymbol{\phi}_{n} \\ &= \sum_{n=1}^{N} \frac{y_{n}-t_{n}}{y_{n}\left(1-y_{n}\right)} \frac{1}{\sqrt{2\pi}}\exp\left(-\frac{a_n^2}{2}\right) \boldsymbol{\phi}_{n} \end{aligned}

次にこれを用いてヘッセ行列\nabla_{\mathbf{w}} \nabla_{\mathbf{w}} E(\mathbf{w})を求める。ベクトルをベクトルで微分するとき、\boldsymbol{\phi}_n\to\boldsymbol{\phi}_n^{\mathrm T}にしておく。

\begin{aligned} \nabla_{\mathbf{w}} \nabla_{\mathbf{w}} E(\mathbf{w}) &=\nabla_{\mathbf{w}} \sum_{n=1}^{N} \frac{y_{n}-t_{n}}{y_{n}\left(1-y_{n}\right)} \mathcal{N}\left(a_{n} \mid 0,1\right) \boldsymbol{\phi}_{n}^{\mathrm{T}} \\ &=\sum_{n=1}^{N}\left\{\left[\nabla_{\mathbf{w}} \frac{y_{n}-t_{n}}{y_{n}\left(1-y_{n}\right)}\right] \mathcal{N}\left(a_{n} \mid 0,1\right) + \frac{y_{n}-t_{n}}{y_{n}\left(1-y_{n}\right)} \left[\nabla_{\mathbf{w}} \mathcal{N}\left(a_{n} \mid 0,1\right)\right]\right\} \boldsymbol{\phi}_{n}^{\mathrm{T}} \\ &=\sum_{n=1}^{N}\left\{\left(\frac{\partial}{\partial y_{n}} \frac{y_{n}-t_{n}}{y_{n}\left(1-y_{n}\right)}\right) \frac{\partial y_{n}}{\partial a_{n}} \nabla_{\mathbf{w}} a_{n} \mathcal{N}\left(a_{n} \mid 0,1\right)+\frac{y_{n}-t_{n}}{y_{n}\left(1-y_{n}\right)}\left(\frac{\partial}{\partial a_{n}} \mathcal{N}\left(a_{n} \mid 0,1\right)\right) \nabla_{\mathbf{w}} a_{n}\right\} \boldsymbol{\phi}_{n}^{\mathrm{T}} \\ &=\sum_{n=1}^{N}\left\{\frac{y_{n}^{2}+t_{n}-2 y_{n} t_{n}}{y_{n}^{2}\left(1-y_{n}\right)^{2}} \mathcal{N}\left(a_{n} \mid 0,1\right)^{\mathrm{2}} \boldsymbol{\phi}_{n}+\frac{y_{n}-t_{n}}{y_{n}\left(1-y_{n}\right)}\left(-a_{n}\right) \mathcal{N}\left(a_{n} \mid 0,1\right) \boldsymbol{\phi}_{n}\right\} \boldsymbol{\phi}_{n}^{\mathrm{T}} \\ &=\sum_{n=1}^{N}\left\{\frac{y_{n}^{2}+t_{n}-2 y_{n} t_{n}}{y_{n}\left(1-y_{n}\right)}\frac{1}{\sqrt{2\pi}}\exp\left(-\frac{a_n^2}{2}\right)-a_{n}\left(y_{n}-t_{n}\right)\right\} \frac{e^{-\frac{a_n^2}{2}}}{\sqrt{2\pi}y_{n}\left(1-y_{n}\right)}\boldsymbol{\phi}_{n}\boldsymbol{\phi}_{n}^{\mathrm{T}} \end{aligned}

公式解答例はeの肩に乗るべき係数が1/2ずつ間違えてる。

演習 4.20

\nabla_{\mathbf{w}_{k}} \nabla_{\mathbf{w}_{j}} E\left(\mathbf{w}_{1}, \ldots, \mathbf{w}_{K}\right)=\sum_{n=1}^{N} y_{n k}\left(I_{k j}-y_{n j}\right) \boldsymbol{\phi}_{n} \boldsymbol{\phi}_{n}^{\mathrm{T}} \tag{4.110}

で定義される多クラスロジスティック回帰問題に対するヘッセ行列が半正定値行列であることを示せ.この問題における非退化ヘッセ行列のサイズは,MK \times MKであることに注意されたい.ここで,Mはパラメータ数であり,Kはクラス数である.半正定値性を証明するため,長さMKの任意のベクトル\mathbf{u}に関して,積\mathbf{u}^{\mathrm{T}}\mathbf{H}\mathbf{u}を考え,イェンセンの不等式を適用する.


※多クラスロジスティック回帰問題でのヘッセ行列は、そのjk成分が(4.110)式で与えられるM\times Mの部分行列から構成される。つまり、

\mathbf{H}=\begin{pmatrix} \mathbf{H}_{11} & \mathbf{H}_{12} & \cdots & \mathbf{H}_{1K} \\ \mathbf{H}_{21} & \mathbf{H}_{22} & \cdots & \mathbf{H}_{2K} \\ \vdots & \vdots & & \vdots \\ \mathbf{H}_{K 1} & \mathbf{H}_{K 2} & \cdots & \mathbf{H}_{KK} \end{pmatrix}, \quad \mathbf{H}_{jk} = \sum_{n=1}^{N}y_{nk}(I_{kj}-y_{nj})\underbrace{\boldsymbol{\phi}_n\boldsymbol{\phi}_n^{\mathrm T}}_{M\times M}

となっている。よって、\mathbf{H}MK \times MKの行列である。

このため、ベクトル\mathbf{u}MK次元である必要があり、M次元ベクトル\mathbf{u}_j(1\le j \le K)について

\mathbf{u} = \begin{pmatrix} \mathbf{u}_{1} \\ \mathbf{u}_{2} \\ \vdots \\ \mathbf{u}_{K} \end{pmatrix}

となる。

演習問題4.15と同じように、任意のベクトル\mathbf{u}について\mathbf{u}^{\mathrm T}\mathbf{Hu} \ge 0であることを示せば良い(半正定値行列なので\ge 0を示す)

要素について注目し計算していくと

\begin{aligned} \mathbf{u}^{\mathrm T}\mathbf{Hu} &= \sum_{k}\sum_{j}\mathbf{u}_{j}^{\mathrm T} \mathbf{H}_{jk}\mathbf{u}_{k} \\ &= \sum_{k}\sum_{j}\mathbf{u}_{j}^{\mathrm T} \sum_n y_{nk}(I_{kj}-y_{nj})\boldsymbol{\phi}_n\boldsymbol{\phi}_n^{\mathrm T} \mathbf{u}_{k} \\ &= \sum_{k}\sum_{j}\sum_n \mathbf{u}_{j}^{\mathrm T} y_{nk}(I_{kj}-y_{nj})\boldsymbol{\phi}_n\boldsymbol{\phi}_n^{\mathrm T} \mathbf{u}_{k} \\ &= \sum_{k}\sum_{j}\sum_n\left( \mathbf{u}_{j}^{\mathrm T} y_{nk}I_{kj}\boldsymbol{\phi}_n\boldsymbol{\phi}_n^{\mathrm T} \mathbf{u}_{k} - \mathbf{u}_{j}^{\mathrm T} y_{nk}y_{nj}\boldsymbol{\phi}_n\boldsymbol{\phi}_n^{\mathrm T} \mathbf{u}_{k} \right) \\ &= \sum_{k}\sum_n\left( \mathbf{u}_{k}^{\mathrm T} y_{nk}\boldsymbol{\phi}_n\boldsymbol{\phi}_n^{\mathrm T} \mathbf{u}_{k} - \sum_{j} \mathbf{u}_{j}^{\mathrm T} y_{nk}y_{nj}\boldsymbol{\phi}_n\boldsymbol{\phi}_n^{\mathrm T} \mathbf{u}_{k} \right) (\because I_{kj}=1\textrm{ when } k=j, \textrm{ otherwise } 0)\\ &= \sum_{k}\sum_n\left\{ y_{nk}(\mathbf{u}_{k}^{\mathrm T} \boldsymbol{\phi}_n)^2 - y_{nk} \sum_{j} y_{nj} \mathbf{u}_{j}^{\mathrm T} \boldsymbol{\phi}_n\boldsymbol{\phi}_n^{\mathrm T} \mathbf{u}_{k} \right\} \\ &= \sum_n\left\{\sum_k y_{nk}(\mathbf{u}_{k}^{\mathrm T} \boldsymbol{\phi}_n)^2 - \sum_k y_{nk} \sum_{j} y_{nj} \sum_{k,j} \mathbf{u}_{j}^{\mathrm T} \boldsymbol{\phi}_n\boldsymbol{\phi}_n^{\mathrm T} \mathbf{u}_{k} \right\} \\ &= \sum_n\left\{\sum_k y_{nk}(\mathbf{u}_{k}^{\mathrm T} \boldsymbol{\phi}_n)^2 - \left(\sum_j y_{nj}\mathbf{u}_{j}^{\mathrm T} \boldsymbol{\phi}_n \right) \left(\sum_k y_{nk}\mathbf{u}_{k}^{\mathrm T} \boldsymbol{\phi}_n \right) \right\} \\ &= \sum_n\left\{\sum_k y_{nk}(\mathbf{u}_{k}^{\mathrm T} \boldsymbol{\phi}_n)^2 - \left( \sum_k y_{nk} \mathbf{u}_{k}^{\mathrm T} \boldsymbol{\phi}_n \right)^2 \right\} \\ \end{aligned}

ここで、0\le y_{nk} \le 1, \sum_{k}y_{nk}=1であり、イェンセンの不等式を凸関数f(x)=x^2に対して適用する。x = \mathbf{u}_{k}^{\mathrm T} \boldsymbol{\phi}_nとすれば

\sum_k y_{nk}(\mathbf{u}_{k}^{\mathrm T} \boldsymbol{\phi}_n)^2 \ge \left( \sum_k y_{nk} \mathbf{u}_{k}^{\mathrm T} \boldsymbol{\phi}_n \right)^2

が成立する。以上から\mathbf{u}^{\mathrm T}\mathbf{Hu} \ge 0となり、多クラスロジスティック回帰問題に対するヘッセ行列\mathbf{H}が半正定値行列であることが示された。

演習 4.21

プロビット関数の逆関数

\Phi(a)=\int_{-\infty}^{a} \mathcal{N}(\theta \mid 0,1) \mathrm{d} \theta \tag{4.114}

とerf関数

\operatorname{erf}(a)=\frac{2}{\sqrt{\pi}} \int_{0}^{a} \exp \left(-\theta^{2}\right) \mathrm{d} \theta \tag{4.115}

\Phi(a)=\frac{1}{2}\left\{1+\operatorname{erf}\left(\frac{a}{\sqrt{2}}\right)\right\} \tag{4.116}

によって関連付けられることを示せ.


\begin{aligned} \mathbf{\Phi}(a) &= \int_{-\infty}^{a} \mathcal{N}(\theta \mid 0,1)d\theta\\ &=\int_{-\infty}^{0} \mathcal{N}(\theta \mid 0,1)d\theta + \int_{0}^{a} \frac{1}{\sqrt{2\pi}}\exp\left(-\frac{\theta^2}{2}\right)d\theta \\ &= \frac{1}{2} + \int_{0}^{a} \frac{1}{\sqrt{2\pi}}\exp\left(-\frac{\theta^2}{2}\right)d\theta \quad \left(\because\quad \int_{-\infty}^{\infty} \mathcal{N}(\theta \mid 0,1)d\theta = 1 \right) \end{aligned}

ここで\displaystyle \frac{\theta}{\sqrt{2}} = xとおくと

\begin{aligned} \mathbf{\Phi}(a) &= \frac{1}{2}+\frac{1}{\sqrt{\pi}}\int_{0}^{\frac{a}{\sqrt{2}}} \exp(-x^2)dx \\ &= \frac{1}{2}\left\{1+\mathrm{erf}\left(\frac{a}{\sqrt{2}}\right)\right\} \end{aligned}

となり、(4.116)式が示された。

演習 4.22

\begin{aligned} Z &=\int f(\mathbf{z}) \mathrm{d} \mathbf{z} \\ & \simeq f\left(\mathbf{z}_{0}\right) \int \exp \left\{-\frac{1}{2}\left(\mathbf{z}-\mathbf{z}_{0}\right)^{\mathrm{T}} \mathbf{A}\left(\mathbf{z}-\mathbf{z}_{0}\right)\right\} \mathrm{d} \mathbf{z} \\ &=f\left(\mathbf{z}_{0}\right) \frac{(2 \pi)^{M / 2}}{|\mathbf{A}|^{1 / 2}} \end{aligned} \tag{4.135}

を使って,ラプラス近似の下での対数モデルエビデンスに対する表現

\ln p(\mathcal{D}) \simeq \ln p\left(\mathcal{D} \mid \theta_{\mathrm{MAP}}\right)+\underbrace{\ln p\left(\theta_{\mathrm{MAP}}\right)+\frac{M}{2} \ln (2 \pi)-\frac{1}{2} \ln |\mathbf{A}|}_{\text {Occam 係数 }} \tag{4.137}

を導出せよ.


P.216参照。(4.136)式を(4.135)式に代入すると

\begin{aligned} p(\mathcal{D}) &=\int p(\mathcal{D} \mid \boldsymbol{\theta}) p(\boldsymbol{\theta}) \mathrm{d} \boldsymbol{\theta} \\ & \simeq p\left(\mathcal{D} \mid \boldsymbol{\theta}_{\mathrm{MAP}}\right) p\left(\boldsymbol{\theta}_{\mathrm{MAP}}\right) \\ & \int \exp \left\{-\frac{1}{2}\left(\boldsymbol{\theta}-\boldsymbol{\theta}_{\mathrm{MAP}}\right) \mathbf{A}\left(\boldsymbol{\theta}-\boldsymbol{\theta}_{\mathrm{MAP}}\right)\right\} \mathrm{d} \boldsymbol{\theta} \\ &=p\left(\mathcal{D} \mid \boldsymbol{\theta}_{\mathrm{MAP}}\right) p\left(\boldsymbol{\theta}_{\mathrm{MAP}}\right) \frac{(2 \pi)^{M / 2}}{|\mathbf{A}|^{1 / 2}} \end{aligned}

これより、両辺の対数を取ると

\ln p(\mathcal{D}) \simeq \ln p\left(\mathcal{D} \mid \boldsymbol{\theta}_{\mathrm{MAP}}\right)+\underbrace{\ln p\left(\theta_{\mathrm{MAP}}\right)+\frac{M}{2} \ln (2 \pi)-\frac{1}{2} \ln |\mathbf{A}|}_{\text {Occam 係数 }} \tag{4.137}

が得られる。

演習 4.23

この演習問題では(4.137)で与えられるモデルエビデンスに対し,ラプラス近似から始めて,BICの結果

\ln p(\mathcal{D}) \simeq \ln p\left(\mathcal{D} \mid \theta_{\mathrm{MAP}}\right)-\frac{1}{2} M \ln N \tag{4.139}

を導出する.パラメータ上での事前確率がp(\theta)=\mathcal{N}\left(\boldsymbol{\theta} \mid \mathbf{m}, \mathbf{V}_{0}\right)形式のガウス分布のとき,ラプラス近似の下での対数モデルエビデンスが以下の式となることを示せ.

\ln p(\mathcal{D}) \simeq \ln p\left(\mathcal{D} \mid \boldsymbol{\theta}_{\mathrm{MAP}}\right)-\frac{1}{2}\left(\boldsymbol{\theta}_{\mathrm{MAP}}-\mathbf{m}\right)^{\mathrm{T}} \mathbf{V}_{0}^{-1}\left(\boldsymbol{\theta}_{\mathrm{MAP}}-\mathbf{m}\right)-\frac{1}{2} \ln |\mathbf{H}|+\text {const.}

ここで,\mathbf{H}\boldsymbol{\theta}_{\mathrm{MAP}}で評価された負の対数尤度\ln p(\mathcal{D}|\boldsymbol{\theta})の2階微分の行列である.事前確率が広い幅を持っている,つまり,\mathbf{V}_{0}^{-1}が小さく,上式右辺第2項が無視できると仮定する.さらに,\mathbf{H}が各データ点に対応する項の和で書けるように独立同時分布(i.i.d)の場合を考える.その場合,対数モデルエビデンスが近似的にBIC表現(4.139)の式で書けることを示せ.


(4.137)式における\mathbf{A}は事後分布の負の対数の2階微分であるヘッセ行列であり、

\mathbf{A} = -\nabla\nabla \ln p(\mathcal{D} \mid \boldsymbol{\theta}_{\mathrm{MAP}})p(\boldsymbol{\theta}_{\mathrm{MAP}}) = -\nabla\nabla\ln p(\boldsymbol{\theta}_{\mathrm{MAP}} \mid \mathcal{D})

である。ここで、問題文から\mathbf{H}\boldsymbol{\theta}_{\mathrm{MAP}}で評価された負の対数尤度\ln p(\mathcal{D} \mid \boldsymbol{\theta}_{\mathrm{MAP}})の2階微分の行列なので

\mathbf{H} = -\nabla\nabla\ln p(\mathcal{D} \mid \boldsymbol{\theta}_{\mathrm{MAP}})

であり、この2式から

\mathbf{A} = \mathbf{H} -\nabla\nabla\ln p(\boldsymbol{\theta}_{\mathrm{MAP}})

となる。今、p(\boldsymbol{\theta}) = \mathcal{N}(\boldsymbol{\theta}\mid \mathbf{m},\mathbf{V}_0)とすると

\begin{aligned} -\nabla\nabla\ln p(\boldsymbol{\theta}_{\mathrm{MAP}}) &= -\nabla\nabla \ln \left[ \frac{1}{(2\pi)^{M/2}|\mathbf{V}_0|^{1/2}} \exp\left\{ -\frac{1}{2} (\boldsymbol{\theta}_{\mathrm{MAP}}-\mathbf{m})^{\mathrm T} \mathbf{V}_0^{-1} (\boldsymbol{\theta}_{\mathrm{MAP}}-\mathbf{m}) \right\} \right] \\ &=\mathbf{V}_0^{-1} \end{aligned}

なので、\mathbf{A} = \mathbf{H} + \mathbf{V}_0^{-1}となる。

問題文の通り、もし事前分布が十分に広い幅を持っていれば(またはデータの数Nが多ければ)\mathbf{V}_0^{-1}\mathbf{H}に比べ十分に小さくなるので\mathbf{A} \simeq \mathbf{H}と近似できる。(4.137)式にこれを代入すれば

\ln p(\mathcal{D}) \simeq \ln p\left(\mathcal{D} \mid \theta_{\mathrm{MAP}}\right)-\frac{1}{2}\left(\theta_{\mathrm{MAP}}-\mathbf{m}\right)^{\mathrm{T}} \mathbf{V}_{0}^{-1}\left(\theta_{\mathrm{MAP}}-\mathbf{m}\right)-\frac{1}{2} \ln |\mathbf{H}|+\text { const. }

が得られる。

再び\boldsymbol{\theta}の事前分布が平坦であるという仮定を用いると、上式の第1項に対して第2項を無視できる。またデータ点がi.i.d.であると仮定すると、ヘッセ行列を各データ点からの寄与\mathbf{H}_nの和で表すことができる。また、その和をデータ点の数Nと各データ点からの寄与の平均値\hat{\mathbf{H}}を用いて表すこともできる。

\mathbf{H} = \sum_{n=1}^N \mathbf{H}_n = N\hat{\mathbf{H}}, \quad \hat{\mathbf{H}} = \frac{1}{N}\sum_{n=1}^N \mathbf{H}_n

すると、\mathbf{H}M \times MM\boldsymbol{\theta}のパラメータ数)の行列で、非退化(フルランク)であると仮定するなら

\ln |\mathbf{H}| = \ln |N\hat{\mathbf{H}}| = \ln(N^M |\hat{\mathbf{H}}|) = M\ln N + \ln |\hat{\mathbf{H}}|

第2項は\ln Nと比較してO(1)なので無視できる(ここよくわからない)ので、最終的に(4.139)

\ln p(\mathcal{D}) \simeq \ln p\left(\mathcal{D} \mid \theta_{\mathrm{MAP}}\right)-\frac{1}{2} M \ln N \tag{4.139}

のように粗く近似することができる。これが ベイズ情報量規準 (BIC) である。

演習 4.24

2.3.2節からの結果を利用して,パラメータ\mathbf{w}の事後確率分布がガウス分布である場合のロジスティック回帰モデルの周辺化に対する結果

p\left(\mathcal{C}_{1} \mid \mathbf{t}\right)=\int \sigma(a) p(a) \mathrm{d} a=\int \sigma(a) \mathcal{N}\left(a \mid \mu_{a}, \sigma_{a}^{2}\right) \mathrm{d} a \tag{4.151}

を導け.


※P.218 4.5.2 予測分布のところで、「デルタ関数は\mathbf{w}に線形制約を科し,\boldsymbol{\phi}に直交するすべての方向に積分することで,同時分布q(\mathbf{w})から周辺分布を形成することに留意して,p(a)を評価することができる.」という記述がある。そこで、M次元空間を回転させ、\boldsymbol{\phi}に平行な\mathbf{w}の成分をw_{\parallel}とし、\boldsymbol{\phi}に直交するM-1個の成分をまとめて\mathbf{w}_{\perp}とする。すなわち

a = \mathbf{w}^{\mathrm T}\boldsymbol{\phi} = w_{\parallel}\|\boldsymbol{\phi}\|,\quad \textrm{where}\ \mathbf{w}^{\mathrm T} = (w_{\parallel}, \mathbf{w}_{\perp})

とする。これを用いて(4.147)式に代入すると

\begin{aligned} \int \sigma\left(\mathbf{w}^{\mathrm{T}} \boldsymbol{\phi}\right) q(\mathbf{w}) \mathrm{d} \mathbf{w} &=\iint \sigma\left(w_{\parallel}\|\boldsymbol{\phi}\|\right) q\left(w_{\parallel}, \mathbf{w}_{\perp}\right) \mathrm{d} w_{\parallel} \mathrm{d} \mathbf{w}_{\perp} \\ &=\int \sigma\left(w_{\parallel}\|\boldsymbol{\phi}\|\right) \int q\left(w_{\parallel}, \mathbf{w}_{\perp} \right) \mathrm{d}\mathbf{w}_{\perp} \mathrm{d} w_{\parallel} \\ &=\int \sigma\left(w_{\parallel}\|\boldsymbol{\phi}\|\right) q\left(w_{\|}\right) \mathrm{d} w_{\parallel} \\ &=\int \sigma(a) q(w_{\|}) \mathrm{d} w_{\parallel} \end{aligned}

ここで、同時確率分布q(w_{\parallel}, \mathbf{w}_{\perp})はガウス分布なので、2.3.2節の内容から、周辺分布q(w_{\parallel})もガウス分布である。すなわち

q(w_{\parallel}) = \mathcal{N}(w_{\parallel}\mid \mu, \sigma^2)

のような形で記述することができる。

上式はw_{\parallel}についてのスカラー量になっているので、行列形式で書くために単位ベクトル

\mathbf{e} = \frac{\boldsymbol{\phi}}{\|\boldsymbol{\phi}\|}

を導入すると、\mu = \mathbf{e}^{\mathrm T}\mathbf{m}_{\mathrm{MAP}}, \sigma^2 = \mathbf{e}^{\mathrm T}\mathbf{S}_{\mathrm{MAP}}\mathbf{e}となるような\mathbf{m}_{\mathrm{MAP}}, \mathbf{S}_{\mathrm{MAP}}が存在する。

q(w_{\parallel}) = \mathcal{N}(w_{\parallel}\mid \mu, \sigma^2) = \|\boldsymbol{\phi}\|\mathcal{N}(a\mid \boldsymbol{\phi}^{\mathrm T}\mathbf{m}_{\mathrm{MAP}}, \boldsymbol{\phi}^{\mathrm T}\mathbf{S}_{\mathrm{MAP}}\boldsymbol{\phi})

また、a = w_{\parallel}\|\boldsymbol{\phi}\|より\|\boldsymbol{\phi}\|\mathrm{d}w_{\parallel} = \mathrm{d}aを利用すると

\begin{aligned} \int \sigma(a) q(w_{\|}) \mathrm{d} w_{\parallel} &= \|\boldsymbol{\phi}\| \int \sigma(a) \mathcal{N}(a\mid \boldsymbol{\phi}^{\mathrm T}\mathbf{m}_{\mathrm{MAP}}, \boldsymbol{\phi}^{\mathrm T}\mathbf{S}_{\mathrm{MAP}}\boldsymbol{\phi})\mathrm{d}w_{\parallel} \\ &= \int \sigma(a) \mathcal{N}(a\mid \boldsymbol{\phi}^{\mathrm T}\mathbf{m}_{\mathrm{MAP}}, \boldsymbol{\phi}^{\mathrm T}\mathbf{S}_{\mathrm{MAP}}\boldsymbol{\phi})\mathrm{d}a \end{aligned}

最後に\mu_a = \boldsymbol{\phi}^{\mathrm T}\mathbf{m}_{\mathrm{MAP}}, \sigma^{2}_{a} = \boldsymbol{\phi}^{\mathrm T}\mathbf{S}_{\mathrm{MAP}}\boldsymbol{\phi}とすれば、(4.151)式が導かれる。

演習 4.25

\sigma(a)=\frac{1}{1+\exp (-a)} \tag{4.59}

で定義されるロジスティックシグモイド関数\sigma(a)をスケールパラメータを持つプロビット関数の逆関数\mathrm{\Phi}(\lambda a)で近似するとする.ここで,\mathrm{\Phi}(a)

\Phi(a)=\int_{-\infty}^{a} \mathcal{N}(\theta \mid 0,1) \mathrm{d} \theta \tag{4.114}

で定義される.2つの関数の微分がa=0で等しいように\lambdaを選ぶ場合,\lambda^2 = \pi /8となることを示せ.


※ロジスティックシグモイド\sigma(a)a=0での微分\displaystyle \left. \frac{d\sigma}{da}\right|_{a=0}とプロビット関数の逆関数\Phi(\lambda a)a=0での微分\displaystyle \left. \frac{d\Phi(\lambda a)}{da}\right|_{a=0}が一致すれば良い。

\left. \frac{d\sigma}{da} \right|_{a=0} = \left. \frac{e^{-a}}{(1+e^{-a})^2}\right|_{a=0} = \frac{1}{4} \tag{1}
\begin{aligned} \left. \frac{d\Phi(\lambda a)}{da}\right|_{a=0} &= \left. \frac{d(\lambda a)}{da} \frac{d}{d(\lambda a)}\int_{-\infty}^{\lambda a} \mathcal{N}(\theta\mid 0, 1)d\theta\right|_{a=0} \\ &= \left. \lambda \mathcal{N}(\lambda a\mid 0, 1)\right|_{a=0} \\ &= \left. \frac{\lambda}{\sqrt{2\pi}} \exp\left( -\frac{(\lambda a)^2}{2} \right)\right|_{a=0} \\ &= \frac{\lambda}{\sqrt{2\pi}} \end{aligned}\tag{2}

(1), (2)が一致するので、

\lambda = \frac{\sqrt{2\pi}}{4},\quad \lambda^2 = \frac{\pi}{8}

となる。

演習 4.26

この演習問題で,ガウス分布とプロビット関数の逆関数のたたみ込みに対する関係

\int \Phi(\lambda a) \mathcal{N}\left(a \mid \mu, \sigma^{2}\right) \mathrm{d} a=\Phi\left(\frac{\mu}{\left(\lambda^{-2}+\sigma^{2}\right)^{1 / 2}}\right) \tag{4.152}

を証明する.この証明を行うため,\muに関する左辺の微分が右辺の微分に等しいことを示せ.また,\muに関して両辺を積分して,積分定数が消えることを示せ.左辺を微分する前に,aでの積分をzでの積分で置き換えるように変数変換a=\mu+\sigma zを導入すれば便利である.関係(4.152)の左辺を微分すれば解析的に評価可能なzに関するガウス分布の積分が得られる.


※右辺の\muについての微分は簡単なのでそちらから求める。

\frac{d}{d \mu} \Phi\left(\frac{\mu}{\left(\lambda^{-2}+\sigma^{2}\right)^{1 / 2}}\right)=\frac{1}{\sqrt{2 \pi}} \frac{1}{\left(\lambda^{-2}+\sigma^{2}\right)^{1 / 2}} \exp \left\{-\frac{\mu^{2}}{2\left(\lambda^{-2}+\sigma^{2}\right)}\right\} \tag{1}

次に左辺について、左辺の式をf(a)とおき、誘導に従ってa = \mu+\sigma zを導入すると

\begin{aligned} f(a)&=\int \Phi(\lambda a) \mathcal{N}\left(a \mid \mu, \sigma^{2}\right) d a \\ &=\int \Phi(\lambda(\mu+\sigma z)) \mathcal{N}\left(\mu+\sigma z \mid \mu, \sigma^{2}\right) \sigma d z \quad (\because da = \sigma dz)\\ &=\int \Phi(\lambda \mu+\lambda \sigma z) \frac{1}{\left(2 \pi \sigma^{2}\right)^{1/2}} \exp \left\{-\frac{1}{2} z^{2}\right\} \sigma d z \end{aligned}

f(a)に対し\muについての微分を実行する。ここで

\frac{\partial}{\partial \mu}\Phi(\lambda(\mu + \sigma z)) = \frac{\partial (\lambda(\mu + \sigma z))}{\partial\mu}\frac{\partial \Phi(\lambda(\mu + \sigma z))}{\partial (\lambda(\mu + \sigma z))} = \lambda \mathcal{N}(\lambda(\mu + \sigma z))

であることに注意すると

\begin{aligned} \frac{\partial f}{\partial \mu} &=\int \lambda \mathcal{N}(\lambda \mu+\lambda \sigma z) \frac{1}{\sqrt{2 \pi}} \exp \left\{-\frac{1}{2} z^{2}\right\} d z \\ &=\frac{\lambda}{2 \pi} \int \exp \left\{-\frac{1}{2} z^{2}-\frac{\lambda^{2}}{2}(\mu+\sigma z)^{2}\right\} d z \end{aligned}

ここで、積分記号の中がガウス分布の形になるようにzについて平方完成を行うと、積分記号の中は

\begin{aligned} -\frac{1}{2}\left(z^{2}+\lambda^{2}(\mu+\sigma z)^{2}\right) &=-\frac{1}{2}\left\{\left(1+\lambda^{2} \sigma^{2}\right) z^{2}+2 \lambda^{2} \mu \sigma z+\lambda^{2} \mu^{2}\right\} \\ &=-\frac{1+\lambda^{2} \sigma^{2}}{2}\left\{\left(z+\frac{\lambda^{2} u \sigma}{1+\lambda^{2} \sigma^{2}}\right)^{2}-\frac{\lambda^{4} \mu^{2} \sigma^{2}}{\left(1+\lambda^{2} \sigma^{2}\right)^{2}}+\frac{\lambda^{2} \mu^{2}}{1+\lambda^{2} \sigma^{2}}\right\} \\ &=-\frac{1+\lambda^{2} \sigma^{2}}{2}\left(z+\frac{\lambda^{2} \mu \sigma}{1+\lambda^{2} \sigma^{2}}\right)^{2}-\frac{\lambda^{2} u^{2}}{2\left(1+\lambda^{2} \sigma^{2}\right)} \end{aligned}

となる。さらに途中でガウス分布の正規化定数\displaystyle \int \exp \left(-\frac{(x-\mu)^{2}}{2 \sigma^{2}}\right) dz = \sqrt{2\pi \sigma^2}を利用することで

\begin{aligned} \frac{\partial f}{\partial \mu} &=\frac{\lambda}{2 \pi} \int \exp \left\{-\frac{1+\lambda^{2} \sigma^{2}}{2}\left(z+\frac{\lambda^{2} \mu \sigma}{1+\lambda^{2} \sigma^{2}}\right)^{2}-\frac{\lambda^{2} u^{2}}{2\left(1+\lambda^{2} \sigma^{2}\right)}\right\} d z \\ &=\frac{\lambda}{2 \pi} \exp \left(-\frac{\lambda^{2} u^{2}}{2\left(1+\lambda^{2} \sigma^{2}\right)}\right) \underbrace{\int \exp \left\{-\frac{1+\lambda^{2} \sigma^{2}}{2}\left(z+\frac{\lambda^{2} \mu \sigma}{1+\lambda^{2} \sigma^{2}}\right)^{2}\right\}}_{\textrm{Gaussian integral }} d z \\ &=\frac{\lambda}{2 \pi} \exp \left(-\frac{\lambda^{2} u^{2}}{2\left(1+\lambda^{2} \sigma^{2}\right)}\right) \sqrt{\frac{2 \pi}{1+\lambda^{2} \sigma^{2}}} \\ &=\frac{\lambda}{\sqrt{2 \pi}} \frac{1}{\left(1+\lambda^{2} \sigma^{2}\right)^{1 / 2}} \exp \left(\frac{-\lambda^{2} \mu^{2}}{2\left(1+\lambda^{2} \sigma^{2}\right)}\right) \\ &=\frac{1}{\sqrt{2 \pi}} \frac{1}{\left(\lambda^{-2}+\sigma^{2}\right)^{1 / 2}} \exp \left\{-\frac{\mu^{2}}{2\left(\lambda^{-2}+\sigma^{2}\right)}\right\} \end{aligned} \tag{2}

(1), (2)の結果から両辺の\muに関する微分が等しいことが示された。

また、(4.152)の両辺について\mu \to -\inftyを考えると両辺とも0となる。これは積分定数が0であることを示し、微分形が一致することと合わせると、(4.152)式の等号が成立することが示された。

Discussion

ChoikoChoiko

いつもお世話になります。超些末な点ではございますが...
演習問題4.2の5行目のtnはtn^T、11行目のφ0=0はφ0=1と思われます。
前者は問題文もそうなってますが、本文182Pの記述とは異なっています。

ChoikoChoiko

いつもお世話になります。超些末な点ではございますが...
演習問題4.10の下から6行目の第2項の符号は+であると思われます。
ご確認いただければ幸いです。

ChoikoChoiko

お世話になります。
あまり自信はないのですが、まず、演習問題4.11のφの要素xの表記はx^Tではないでしょうか?
ちなみにこのベクトルxは要素数がL個あり、いずれか一つが1、他は0のベクトルという理解はあってますでしょうか?
よろしければ、xベクトルの要素についても記載していただくとありがたいです。

ChoikoChoiko

ありがとうございます。
詳しい解答にいつも助けられております。
お手間おかけしました。

ChoikoChoiko

お世話になります。
演習4.15についてです。
”Φがフルランクである”という点は、テキストおよび本問以前の演習問題で触れられていないと思いますので、可能であれば、解説を加えていただけると大変助かります。
いかがでしょうか?

ChoikoChoiko

お世話になります。
演習4.26についてです。
解答部分の3行目のa = μ + σλは、a = μ + σzだと思われます。
超超些末な点ですので、YOSHITAKA先生が超お手隙の時にでも確認していただけたらと思います。